Доказательство теоремы фалеса с рисунком: МАТВОКС — Теорема о пропорциональных отрезках. Обобщенная теорема Фалеса

Содержание

формула и примеры решения задач

Содержание:

Формулировка теоремы Фалеса

Теорема

Если на одной из двух прямых отложить последовательно несколько отрезков и через их концы провести параллельные прямые, пересекающие вторую прямую, то они отсекут на второй прямой пропорциональные отрезки (рис. 1).

В теореме нет ограничений на взаимное расположение секущих (она верна как для пересекающихся прямых, так и для параллельных). Также не важно, где находятся отрезки на секущих.

Теорема

Обобщённая теорема Фалеса

Параллельные прямые отсекают на секущих пропорциональные отрезки (рис. 1):

$$\frac{A_{1} A_{2}}{B_{1} B_{2}}=\frac{A_{2} A_{3}}{B_{2} B_{3}}=\frac{A_{1} A_{3}}{B_{1} B_{3}}$$

Теорема Фалеса является частным случаем обобщённой теоремы Фалеса, поскольку равные отрезки можно считать пропорциональными отрезками с коэффициентом пропорциональности, равным 1.

Теорема

Обратная теорема Фалеса

Если прямые, пересекающие две другие прямые (параллельные или нет), отсекают на обеих из них равные (или пропорциональные) между собой отрезки, начиная от вершины, то такие прямые параллельны (рис. 2).

Замечание. В обратной теореме Фалеса важно, что равные отрезки начинаются от вершины.

Примеры решения задач

Пример

Задание. Разделить данный отрезок на четыре равные части.

Решение. Пусть $AB$ — заданный отрезок (рис. 3), который необходимо разделить на четыре равные части.

Через точку $A$ проведем произвольную полупрямую $a$ и отложим на ней последовательно четыре равных между собой отрезка $AC, CD, DE, EK$ .

Соединим точки $B$ и $K$ отрезком и проведем через оставшиеся точки $C$, $D$ и $E$ прямые, параллельные прямой $BK$ так, чтобы они пересекли отрезок $AB$ .

Согласно теореме Фалеса отрезок $AB$ разделится на четыре равные части.

Слишком сложно?

Теорема Фалеса не по зубам? Тебе ответит эксперт через 10 минут!

Пример

Задание. На стороне $AB$ треугольника $ABC$ отмечена точка $K$. Отрезок $CK$ пересекает медиану $AM$ треугольника в точке $P$, причем $AK = AP$. Найти отношение $BK : PM$ .

Решение. Проведем через точку $M$ прямую, параллельную $CK$, которая пересечет $AB$ в точке $D$ (рис. 4).

По теореме Фалеса $BD = KD$ .

По теореме о пропорциональных отрезках имеем, что

$$P M=K D=\frac{B K}{2} \Rightarrow B K: P M=2: 1$$

Ответ. $B K: P M=2: 1$

Историческая справка

Теорема Фалеса (а также теоремы Чевы и Менелая) применяются в первую очередь тогда, когда в задаче даны соотношения между отрезками. Очень часто при этом приходится проводить дополнительный отрезок.

Аргентинская музыкальная группа представила песню, посвящённую теореме. В видеоклипе для этой песни приводится доказательство для прямой теоремы для пропорциональных отрезков.

Теорема Фалеса до сих пор используется в морской навигации в качестве правила о том, что столкновение судов, двигающихся с постоянной скоростью, неизбежно, если сохраняется курс судов друг на друга.

Вне русскоязычной литературы теоремой Фалеса иногда называют другую теорему планиметрии, а именно, утверждение о том, что вписанный угол, опирающийся на диаметр окружности, является прямым. Открытие этой теоремы действительно приписывается Фалесу, о чём есть свидетельство Прокла.

Как доказать теорему фалеса 8. Теорема Фалеса

Если стороны угла, пересекают прямые параллельные линии которые одну из сторон разделяют на несколько отрезков, то и вторую сторону, прямые так же разделят на равнозначны с другой стороной отрезки.

Теорему Фалеса доказывает следующее: С 1 , С 2 , С 3 — это места где пересекаются прямые параллельные на любой стороне угла. С 2 находится посередине относительно С 1 и С 3 .. Точки D 1 , D 2 , D 3 — это места где пересекаются прямые, которые соответствуют прямым с другой стороной угла. Доказываем, что когда C 1 C 2 = C 2 C з, значит и D 1 D 2 =D 2 D 3 .
Проводим в месте D 2 прямой отрезок КР, параллельный участку C 1 C 3 . В свойствах параллелограмма C 1 C 2 =KD 2 , C 2 C 3 = D 2 P. Если C 1 C 2 =C 2 C 3 , то и KD 2 =D 2 P.

Полученные треугольные фигуры D 2 D 1 K и D 2 D 3 P равняются. И D 2 K=D 2 P по доказательству. Углы с верхней точкой D 2 равняются как вертикальные, а углы D 2 KD 1 и D 2 PD 3 равняются как внутренние накрест лежащие при параллельных C 1 D 1 и C 3 D 3 и разделяющей KP.

Так как D 1 D 2 =D 2 D 3 теорема доказана по равенству сторон треугольника

Заметка:
Если взять не стороны угла, а два прямых отрезка, доказательство будет такое же.
Любые прямые отрезки параллельные друг другу, которые пересекают две рассматриваемые нами прямые и разделяющие одну из них на одинаковые участки, тоже самое делают и со второй.

Рассмотрим несколько примеров

Первый пример

Условием задания требуется разбить прямую СD на п одинаковых отрезков.
Проводим от точки С полу-прямую с, которая не лежит на прямой СD. Отметим на ней одинаковые по величине части. СС 1 , С 1 С 2 , С 2 С 3 …..С п-1 С п. Соединяем С п с D. Проводим прямые от точек С 1 ,С 2 ,….,С п-1 которые будут параллельны относительно С п D. Прямые будут пересекать СD в местах D 1 D 2 D п-1 и разделять прямую СD на п одинаковых отрезков.

Второй пример

На стороне АВ треугольника АВС отмечена точка СК. Отрезок СК пересекает медиану АМ треугольника в точке Р, при этом АК= АР. Требуется найти отношение ВК к РМ.
Проводим через точку М прямой отрезок, параллельный СК, который пересекает АВ в точке D

По теореме Фалеса ВD=КD
По теореме пропорциональных отрезков получаем, что
РМ = КD = ВК/2, следовательно, ВК: РМ = 2:1
Ответ: ВК: РМ = 2:1

Третий пример

В треугольнике АВС, сторона ВС = 8 см. Прямая DE пересекает стороны АВ и ВС параллельно АС. И отсекает на стороне ВС отрезок ЕС = 4см. Доказать, что АD = DВ.

Так как ВС = 8 см и ЕС = 4см, то
ВЕ = ВС-ЕС, следовательно, ВЕ = 8-4 = 4(см)
По теореме Фалеса , так как АС параллельна DE и ЕС = ВЕ то, следовательно, АD = DВ. Что и требовалось доказать.

В женском журнале — онлайн, Вы найдете много интересной информации для себя. Так же есть раздел, посвященный стихам которые написал Сергей Есенин . Заходите не пожалеете!

Если параллельные прямые, пересекающие стороны угла, отсекают на одной его стороне равные отрезки, то они отсекают равные отрезки и на другой его стороне.

Доказательство. Пусть А 1 , А 2 , А 3 — точки пересечения параллельных прямых с одной из сторон угла и А 2 лежит между А 1 и А 3 (рис.1).

Пусть B 1 В 2 , В 3 — соответствующие точки пересечения этих прямых с другой стороной угла. Докажем, что если А 1 А 2 = A 2 A 3 , то В 1 В 2 = В 2 В 3 .

Проведем через точку В 2 прямую EF, параллельную прямой А 1 А 3 . По свойству параллелограмма А 1 А 2 = FB 2 , A 2 A 3 = B 2 E .

И так как А 1 А 2 = A 2 A 3 , то FB 2 = В 2 Е.

Треугольники B 2 B 1 F и В 2 В 3 Е равны по второму признаку. У них B 2 F = В 2 Е по доказанному. Углы при вершине В 2 равны как вертикальные, а углы B 2 FB 1 и B 2 EB 3 равны как внутренние накрест лежащие при параллельных А 1 В 1 и A 3 B 3 и секущей EF. Из равенства треугольников следует равенство сторон: В 1 В 2 = В 2 В 3 . Теорема доказана.

С использованием теоремы Фалеса устанавливается следующая теорема.

Теорема 2. Средняя линия треугольника параллельна третьей стороне и равна ее половине.

Средней линией треугольника называется отрезок, соединяющий середины двух его сторон. На рисунке 2 отрезок ED — средняя линия треугольника ABC.

ED — средняя линия треугольника ABC

Пример 1. Разделить данный отрезок на четыре равные части.

Решение. Пусть АВ — данный отрезок (рис.3), который надо разделить на 4 равные части.

Деление отрезка на четыре равные части

Для этого через точку А проведем произвольную полупрямую а и отложим на ней последовательно четыре равных между собой отрезка AC, CD, DE, ЕК.

Соединим точки В и К отрезком. Проведем через оставшиеся точки С, D, Е прямые, параллельные прямой ВК, так, чтобы они пересекли отрезок АВ.

Согласно теореме Фалеса отрезок АВ разделится на четыре равные части.

Пример 2. Диагональ прямоугольника равна а. Чему равен периметр четырехугольника, вершины которого являются серединами сторон прямоугольника?

Решение. Пусть условию задачи отвечает рисунок 4.

Тогда EF — средняя линия треугольника ABC и, значит, по теореме 2. $$ EF = \frac{1}{2}AC = \frac{a}{2} $$

Аналогично $$ HG = \frac{1}{2}AC = \frac{a}{2} , EH = \frac{1}{2}BD = \frac{a}{2} , FG = \frac{1}{2}BD = \frac{a}{2} $$ и, следовательно, периметр четырехугольника EFGH равен 2a.

Пример 3. Стороны треугольника равны 2 см, 3 см и 4 см, а вершины его — середины сторон другого треугольника.

Найти периметр большого треугольника.

Решение. Пусть условию задачи отвечает рисунок 5.

Отрезки АВ, ВС, АС — средние линии треугольника DEF. Следовательно, согласно теореме 2 $$ AB = \frac{1}{2}EF\ \ ,\ \ BC = \frac{1}{2}DE\ \ ,\ \ AC = \frac{1}{2}DF $$ или $$ 2 = \frac{1}{2}EF\ \ ,\ \ 3 = \frac{1}{2}DE\ \ ,\ \ 4 = \frac{1}{2}DF $$ откуда $$ EF = 4\ \ ,\ \ DE = 6\ \ ,\ \ DF = 8 $$ и, значит, периметр треугольника DEF равен 18 см.

Пример 4. В прямоугольном треугольнике через середину его гипотенузы проведены прямые, параллельные его катетам. Найти периметр образовавшегося прямоугольника, если катеты треугольника равны 10 см и 8 см.

Решение. В треугольнике ABC (рис.6)

∠ А прямой, АВ = 10 см, АС = 8 см, KD и MD — средние линии треугольника ABC, откуда $$ KD = \frac{1}{2}AC = 4 см. \\ MD = \frac{1}{2}AB = 5 см. $$ Периметр прямоугольника К DMА равен 18 см.

О параллельных и секущих.

Вне русскоязычной литературы теоремой Фалеса иногда называют другую теорему планиметрии, а именно, утверждение о том , что вписанный угол , опирающийся на диаметр окружности , является прямым. Открытие этой теоремы действительно приписывается Фалесу, о чём есть свидетельство Прокла .

Формулировки

Если на одной из двух прямых отложить последовательно несколько равных отрезков и через их концы провести параллельные прямые, пересекающие вторую прямую, то они отсекут на второй прямой равные отрезки.

Более общая формулировка, также называемая теорема о пропорциональных отрезках

Параллельные прямые отсекают на секущих пропорциональные отрезки :

A 1 A 2 B 1 B 2 = A 2 A 3 B 2 B 3 = A 1 A 3 B 1 B 3 . {\displaystyle {\frac {A_{1}A_{2}}{B_{1}B_{2}}}={\frac {A_{2}A_{3}}{B_{2}B_{3}}}={\frac {A_{1}A_{3}}{B_{1}B_{3}}}.}

Замечания

  • В теореме нет ограничений на взаимное расположение секущих (она верна как для пересекающихся прямых, так и для параллельных). Также не важно, где находятся отрезки на секущих.
  • Теорема Фалеса является частным случаем теоремы о пропорциональных отрезках, поскольку равные отрезки можно считать пропорциональными отрезками с коэффициентом пропорциональности, равным 1.

Доказательство в случае секущих

Рассмотрим вариант с несвязанными парами отрезков: пусть угол пересекают прямые A A 1 | | B B 1 | | C C 1 | | D D 1 {\displaystyle AA_{1}||BB_{1}||CC_{1}||DD_{1}} и при этом A B = C D {\displaystyle AB=CD} .

  1. Проведём через точки A {\displaystyle A} и C {\displaystyle C} прямые, параллельные другой стороне угла. A B 2 B 1 A 1 {\displaystyle AB_{2}B_{1}A_{1}} и C D 2 D 1 C 1 {\displaystyle CD_{2}D_{1}C_{1}} . Согласно свойству параллелограмма: A B 2 = A 1 B 1 {\displaystyle AB_{2}=A_{1}B_{1}} и C D 2 = C 1 D 1 {\displaystyle CD_{2}=C_{1}D_{1}} .
  2. Треугольники △ A B B 2 {\displaystyle \bigtriangleup ABB_{2}} и △ C D D 2 {\displaystyle \bigtriangleup CDD_{2}} равны на основании второго признака равенства треугольников

Доказательство в случае параллельных прямых

Проведем прямую BC . Углы ABC и BCD равны как внутренние накрест лежащие при параллельных прямых AB и CD и секущей BC , а углы ACB и CBD равны как внутренние накрест лежащие при параллельных прямых AC и BD и секущей BC . Тогда по второму признаку равенства треугольников треугольники ABC и DCB равны. Отсюда следует, что AC = BD и AB = CD . ■

Вариации и обобщения

Обратная теорема

Если в теореме Фалеса равные отрезки начинаются от вершины (часто в школьной литературе используется такая формулировка), то обратная теорема также окажется верной. Для пересекающихся секущих она формулируется так:

Таким образом (см. рис.) из того, что C B 1 C A 1 = B 1 B 2 A 1 A 2 = … {\displaystyle {\frac {CB_{1}}{CA_{1}}}={\frac {B_{1}B_{2}}{A_{1}A_{2}}}=\ldots } , следует, что A 1 B 1 | | A 2 B 2 | | … {\displaystyle A_{1}B_{1}||A_{2}B_{2}||\ldots } .

Если секущие параллельны, то необходимо требовать равенство отрезков на обеих секущих между собой, иначе данное утверждение становится неверным (контрпример — трапеция, пересекаемая линией, проходящей через середины оснований).

Этой теоремой пользуются в навигации: столкновение судов, двигающихся с постоянной скоростью, неизбежно, если сохраняется направление с одного судна на другое.

Лемма Соллертинского

Следующее утверждение, двойственно к лемме Соллертинского :

Пусть f {\displaystyle f} — проективное соответствие между точками прямой l {\displaystyle l} и прямой m {\displaystyle m} . Тогда множество прямых X f (X) {\displaystyle Xf(X)} будет множеством касательных к некоторому

План:


Введение

Эта теорема о параллельных прямых. Об угле, опирающемся на диаметр, см. другую теорему.

Теорема Фалеса — одна из теорем планиметрии.

В теореме нет ограничений на взаимное расположение секущих (она верна как для пересекающихся прямых, так и для параллельных). Также неважно, где находятся отрезки на секущих.

Доказательство в случае секущих

Доказательство теоремы Фалеса

Рассмотрим вариант с несвязанными парами отрезков: пусть угол пересекают прямые A A 1 | | B B 1 | | C C 1 | | D D 1 и при этом A B = C D .

Доказательство в случае параллельных прямых

Проведем прямую BC. Углы ABC и BCD равны как внутренние накрест лежащие при параллельных прямых AB и CD и секущей BC, а углы ACB и CBD равны как внутренние накрест лежащие при параллельных прямых AC и BD и секущей BC. Тогда по первому признаку равенства треугольников треугольники ABC и DCB равны. Отсюда следует, что AC = BD и AB = CD. ■

Также существует обобщённая теорема Фалеса :

Параллельные прямые отсекают на секущих пропорциональные отрезки:

Теорема Фалеса является частным случаем обобщённой теоремы Фалеса, поскольку равные отрезки можно считать пропорциональными отрезками с коэффициентом пропорциональности, равным 1.


1. Обратная теорема

Если в теореме Фалеса равные отрезки начинаются от вершины (часто в школьной литературе используется такая формулировка), то обратная теорема также окажется верной. Для пересекающихся секущих она формулируется так:

В обратной теореме Фалеса важно, что равные отрезки начинаются от вершины

Таким образом (см. рис.) из того, что следует, что прямые .

Если секущие параллельны, то необходимо требовать равенство отрезков на обеих секущих между собой, иначе данное утверждение становится неверным (контрпример — трапеция, пересекаемая линией, проходящей через середины оснований).


2. Теорема Фалеса в культуре

Аргентинская музыкальная группа Les Luthiers (исп. ) представила песню, посвящённую теореме. В видеоклипе для этой песни приводится доказательство для прямой теоремы для пропорциональных отрезков.

3. Интересные факты

  • Теорема Фалеса до сих пор используется в морской навигации в качестве правила о том, что столкновение судов, двигающихся с постоянной скоростью, неизбежно, если сохраняется курс судов друг на друга.
  • Вне русскоязычной литературы теоремой Фалеса иногда называют другую теорему планиметрии, а именно, утверждение о том, что вписанный угол, опирающийся на диаметр окружности, является прямым. Открытие этой теоремы действительно приписывается Фалесу, о чём есть свидетельство Прокла.
  • Основы геометрии Фалес постигал в Египте .

Примечания

  1. El Teorema de Thales por Les Luthiers en You Tube — www.youtube.com/watch?v=czzj2C4wdxY
  2. 3. Путешествие в Египет / Главная / Античная литература и философия. Фалес из Милета — www.fales-iz-mileta.narod.ru/3_puteshestvie_v_egipet
скачать
Данный реферат составлен на основе статьи из русской Википедии . Синхронизация выполнена 16.07.11 23:06:34
Похожие рефераты:

Теорема фалеса и ее доказательство. Теорема Фалеса

Если параллельные прямые, пересекающие стороны угла, отсекают на одной его стороне равные отрезки, то они отсекают равные отрезки и на другой его стороне.

Доказательство. Пусть А 1 , А 2 , А 3 — точки пересечения параллельных прямых с одной из сторон угла и А 2 лежит между А 1 и А 3 (рис.1).

Пусть B 1 В 2 , В 3 — соответствующие точки пересечения этих прямых с другой стороной угла. Докажем, что если А 1 А 2 = A 2 A 3 , то В 1 В 2 = В 2 В 3 .

Проведем через точку В 2 прямую EF, параллельную прямой А 1 А 3 . По свойству параллелограмма А 1 А 2 = FB 2 , A 2 A 3 = B 2 E .

И так как А 1 А 2 = A 2 A 3 , то FB 2 = В 2 Е.

Треугольники B 2 B 1 F и В 2 В 3 Е равны по второму признаку. У них B 2 F = В 2 Е по доказанному. Углы при вершине В 2 равны как вертикальные, а углы B 2 FB 1 и B 2 EB 3 равны как внутренние накрест лежащие при параллельных А 1 В 1 и A 3 B 3 и секущей EF. Из равенства треугольников следует равенство сторон: В 1 В 2 = В 2 В 3 . Теорема доказана.

С использованием теоремы Фалеса устанавливается следующая теорема.

Теорема 2. Средняя линия треугольника параллельна третьей стороне и равна ее половине.

Средней линией треугольника называется отрезок, соединяющий середины двух его сторон. На рисунке 2 отрезок ED — средняя линия треугольника ABC.

ED — средняя линия треугольника ABC

Пример 1. Разделить данный отрезок на четыре равные части.

Решение. Пусть АВ — данный отрезок (рис.3), который надо разделить на 4 равные части.

Деление отрезка на четыре равные части

Для этого через точку А проведем произвольную полупрямую а и отложим на ней последовательно четыре равных между собой отрезка AC, CD, DE, ЕК.

Соединим точки В и К отрезком. Проведем через оставшиеся точки С, D, Е прямые, параллельные прямой ВК, так, чтобы они пересекли отрезок АВ.

Согласно теореме Фалеса отрезок АВ разделится на четыре равные части.

Пример 2. Диагональ прямоугольника равна а. Чему равен периметр четырехугольника, вершины которого являются серединами сторон прямоугольника?

Решение. Пусть условию задачи отвечает рисунок 4.

Тогда EF — средняя линия треугольника ABC и, значит, по теореме 2. $$ EF = \frac{1}{2}AC = \frac{a}{2} $$

Аналогично $$ HG = \frac{1}{2}AC = \frac{a}{2} , EH = \frac{1}{2}BD = \frac{a}{2} , FG = \frac{1}{2}BD = \frac{a}{2} $$ и, следовательно, периметр четырехугольника EFGH равен 2a.

Пример 3. Стороны треугольника равны 2 см, 3 см и 4 см, а вершины его — середины сторон другого треугольника. Найти периметр большого треугольника.

Решение. Пусть условию задачи отвечает рисунок 5.

Отрезки АВ, ВС, АС — средние линии треугольника DEF. Следовательно, согласно теореме 2 $$ AB = \frac{1}{2}EF\ \ ,\ \ BC = \frac{1}{2}DE\ \ ,\ \ AC = \frac{1}{2}DF $$ или $$ 2 = \frac{1}{2}EF\ \ ,\ \ 3 = \frac{1}{2}DE\ \ ,\ \ 4 = \frac{1}{2}DF $$ откуда $$ EF = 4\ \ ,\ \ DE = 6\ \ ,\ \ DF = 8 $$ и, значит, периметр треугольника DEF равен 18 см.

Пример 4. В прямоугольном треугольнике через середину его гипотенузы проведены прямые, параллельные его катетам. Найти периметр образовавшегося прямоугольника, если катеты треугольника равны 10 см и 8 см.

Решение. В треугольнике ABC (рис.6)

∠ А прямой, АВ = 10 см, АС = 8 см, KD и MD — средние линии треугольника ABC, откуда $$ KD = \frac{1}{2}AC = 4 см. \\ MD = \frac{1}{2}AB = 5 см. $$ Периметр прямоугольника К DMА равен 18 см.


          1. Формулировка;

          2. Доказательство;

  1. Теорема о пропорциональных отрезках;

  2. Теорема Чевы;

          1. Формулировка;

          2. Доказательство;

  1. Теорема Менелая;

          1. Формулировка;

          2. Доказательство;

  1. Задачи и их решения;

  2. Заключение;

  3. Список использованных источников и литературы.

Введение.

Все незначительное нужно,

Чтобы значительному быть…

И. Северянин
Данный реферат посвящен применению метода параллельных прямых к доказательству теорем и решению задач. Почему мы обращаемся к этому методу? В этом учебном году на школьной олимпиаде по математике была предложена геометрическая задача, которая нам показалась очень сложной. Именно данная задача и дала импульс к началу работы по изучению и освоению метода параллельных прямых при решении задач на нахождение отношения длин отрезков.

Идея самого метода построена на использовании обобщенной теоремы Фалеса. Теорема Фалеса изучается в восьмом классе, ее обобщение и тема «Подобие фигур» в девятом и только в десятом классе, в ознакомительном плане, изучаются две важные теоремы Чевы и Менелая, с помощью которых относительно легко решается ряд задач на нахождение отношения длин отрезков. Поэтому на ступени основного образования мы можем решать довольно узкий круг задач по данному учебному материалу. Хотя на итоговой аттестации за курс основной школы и на ЕГЭ по математике задачи по данной теме (Теорема Фалеса. Подобие треугольников, коэффициент подобия. Признаки подобия треугольников) предлагаются во второй части экзаменационной работы и относятся к высокому уровню сложности.

В процессе работы над рефератом стало возможным углубление наших знаний по данной теме. Доказательство теоремы о пропорциональных отрезках в треугольнике (теорема не входит в школьную программу) построено на методе параллельных прямых. В свою очередь, данная теорема позволила предложить еще один способ доказательства теорем Чевы и Менелая. И в итоге мы смогли научиться решать более широкий круг задач на сравнение длин отрезков. В этом и заключается актуальность нашей работы.

Обобщенная теорема Фалеса.

Формулировка:

Параллельные прямые, пересекающие две данные прямые, отсекают на этих прямых пропорциональные отрезки.
Дано:

Прямая а рассечена параллельными прямыми (А 1 В 1 , А 2 В 2 , А 3 В 3 ,…, А n B n ) на отрезки А 1 А 2 , А 2 А 3 , …, A n -1 A n , а прямая b на отрезки В 1 В 2 , В 2 В 3 , …, В n -1 В n .


Доказать:

Доказательство:

Докажем, например, что

Рассмотрим два случая:

1 случай (рис. б)

Прямые a и b параллельны. Тогда четырехугольники

А 1 А 2 В 2 В 1 и А 2 А 3 В 3 В 2 – параллелограммы. Поэтому

А 1 А 2 = В 1 В 2 и А 2 А 3 =В 2 В 3 , откуда следует, что


2 случай (рис. в)

Прямые a и b не параллельны. Через точку А 1 проведем прямую с , параллельную прямой b . Она пересечет прямые А 2 В 2 и А 3 В 3 в некоторых точках С 2 и С 3 . Треугольники А 1 А 2 С 2 и А 1 А 3 С 3 подобны по двум углам (угол А 1 – общий, углы А 1 А 2 С 2 и А 1 А 3 С 3 равны как соответственные при параллельных прямых А 2 В 2 и А 3 В 3 секущей А 2 А 3 ), поэтому

1+

Или по свойству пропорций

С другой стороны, по доказанному в первом случае имеем А 1 С 2 = В 1 В 2 , С 2 С 3 = В 2 В 3 . Заменяя в пропорции (1) А 1 С 2 на В 1 В 2 и С 2 С 3 на В 2 В 3 , приходим к равенству

что и требовалось доказать.
Теорема о пропорциональных отрезках в треугольнике.

На сторонах АС и ВС треугольника АВС отмечены точки К и М так, что АК:КС= m : n , BM : MC = p : q . Отрезки АМ и ВК пересекаются в точке О (рис. 124б).


Доказать:

Доказательство:
Через точку М проведем прямую MD (рис. 124а), параллельную ВК . Она пересекает сторону АС в точке D , и согласно обобщению теоремы Фалеса

Пусть АК= mx . Тогда в соответствии с условием задачи КС= nx , а так как KD : DC = p : q , то Снова воспользуемся обобщением теоремы Фалеса:

Аналогично доказывается, что .

Теорема Чевы.
Теорема названа в честь итальянского математика Джованни Чевы, который доказал её в 1678 году.

Формулировка:

Если на сторонах АВ, ВС и СА треугольника АВС взяты соответственно точки С 1 , А 1 и В 1 , то отрезки АА 1 , ВВ 1 и СС 1 пересекаются в одной точке тогда и только тогда, когда


Дано:

Треугольник АВС и на его сторонах АВ , ВС и АС отмечены точки С 1 , А 1 и В 1 .


Доказать:

2.отрезки А А 1 , ВВ 1 и СС 1 пересекаются в одной точке.


Доказательство:
1. Пусть отрезки АА 1 , ВВ 1 и СС 1 пересекаются в одной точке О . Докажем, что выполнено равенство (3). По теореме о пропорциональных отрезках в треугольнике 1 имеем:

Левые части этих равенств одинаковы, значит, равны и правые части. Приравнивая их, получаем


Разделив обе части на правую часть, приходим к равенству (3).

2. Докажем обратное утверждение. Пусть точки С 1 , А 1 и В 1 взяты на сторонах АВ , ВС и СА так, что выполнено равенство (3). Докажем, что отрезки АА 1 , ВВ 1 и СС 1 пересекаются в одной точке. Обозначим буквой О точку пересечения отрезков А А 1 и ВВ 1 и проведем прямую СО . Она пересекает сторону АВ в некоторой точке, которую обозначим С 2 . Так как отрезки АА 1 , ВВ 1 и СС 1 пересекаются в одной точке, то по доказанному в первом пункте

Итак, имеют место равенства (3) и (4).

Сопоставляя их, приходим к равенству = , которое показывает, что точки C 1 и C 2 делят сторону AB C 1 и C 2 совпадают, и, значит, отрезки АА 1 , ВВ 1 и СС 1 пересекаются в точке O .

Что и требовалось доказать.
Теорема Менелая.

Формулировка:

Если на сторонах АВ и ВС и продолжении стороны АС (либо на продолжениях сторон АВ, ВС и АС) взяты соответственно точки С 1 , А 1 , В 1 , то эти точки лежат на одной прямой тогда и только тогда, когда

Дано:

Треугольник АВС и на его сторонах АВ , ВС и АС отмечены точки С 1 , А 1 и В 1 .


Доказать:


2. точки А 1 ,С 1 и В 1 лежат на одной прямой
Доказательство:
1. Пусть точки А 1 ,С 1 и В 1 лежат на одной прямой. Докажем, что выполнено равенство (5). Проведем AD ,BE и CF параллельно прямой В 1 А 1 (точка D лежит на прямой ВС ). Согласно обобщенной теоремы Фалеса имеем:


Перемножая левые и правые части этих равенств, получаем


т.е. выполнено равенство (5).
2. Докажем обратное утверждение. Пусть точка В 1 взята на продолжении стороны АС , а точки С 1 и А 1 – на сторонах АВ и ВС , причем так, что выполнено равенство (5). Докажем, что точки А 1 ,С 1 и В 1 лежат на одной прямой. Пусть прямая А 1 С 1 пересекает продолжение стороны АС в точке В 2, тогда по доказанному в первом пункте

Сопоставляя (5) и (6), приходим к равенству = , которое показывает, что точки В 1 и В 2 делят сторону АС в одном и том же отношении. Следовательно, точки В 1 и В 2 совпадают, и, значит, точки А 1 ,С 1 и В 1 лежат на одной прямой. Аналогично доказывается обратное утверждение в случае, когда все три точки А 1 ,С 1 и В 1 лежат на продолжениях соответствующих сторон.

Что и требовалось доказать.

Решение задач.

Предлагается рассмотреть ряд задач на пропорциональное деление отрезков в треугольнике. Как было отмечено выше, для определения места расположения нужных в задаче точек существует несколько методов. В своей работе мы остановились на методе параллельных прямых. Теоретической основой данного метода является обобщенная теорема Фалеса, которая позволяет с помощью параллельных прямых переносить известные отношения пропорции с одной стороны угла на вторую его сторону, таким образом, нужно только удобным для решения задачи способом провести эти параллельные прямые.
Рассмотрим конкретные задачи:
Задача №1 В треугольнике АВС на стороне ВС взята точка М так, что ВМ:МС=3:2. Точка Р делит отрезок АМ в отношении 2:1. Прямая ВР пересекает сторону АС в точке В 1 . В каком отношении точка В 1 делит сторону АС?

Решение : Нужно найти отношение АВ 1:В 1 С, АС искомый отрезок на котором лежит точка В 1 .

Метод параллельных заключается в следующем:


  1. рассечь искомый отрезок параллельными прямыми. Одна ВВ 1 уже есть, а вторую МN проведем через точку М, параллельно ВВ 1 .

  2. Перенести известное отношение с одной стороны угла на другую его сторону, т.е. рассмотреть углы стороны, которых и рассекаются этими прямыми.
Стороны угла С рассекаются прямыми ВВ 1 и МN и по обобщенной теореме Фалеса заключаем В 1 N =3р , NC=2р. Стороны угла МАС пересекают прямые РВ 1 и МN и делят его стороны в отношении 2:1, следовательно АВ 1:В 1 N=2:1 и значит АВ 1 =2n, В 1 N = n . Так как В 1 N =3р , и В 1 N = n , то 3р= n .

Перейдем к интересующему нас отношению АВ 1:В 1 С= АВ 1:(В 1 N+ NС)= 2n:(3р+2р)=(2*3р):(5р)=6:5.

Ответ: АВ 1:В 1 С = 6:5.

Замечание : Данную задачу можно было решить, используя теорему Менелая. Применив ее к треугольнику АМС. Тогда прямая ВВ 1 пересекает две стороны треугольника в точках В 1 и Р, а продолжение третьей в точке В. Значит применимо равенство: , следовательно
Задача №2 В треугольнике АВС АN – медиана. На стороне АС взята точка М так, что АМ: МС = 1: 3. Отрезки AN и ВМ пересекаются в точке О, а луч СО пересекает АВ в точке К. В каком отношении точка К делит отрезок АВ.

Решение: Нужно найти отношение АК к КВ.

1) Проведем прямую NN 1 параллельную прямой СК и прямую NN 2 параллельную прямой ВМ.

2) Стороны угла АВС пересекаются прямыми СК и NN 1 и по обобщенной теореме Фалеса заключаем ВN 1:N 1 K=1:1 или ВN 1 = N 1 K = y .

3) Стороны угла ВСM пересекаются прямыми BM и NN 2 и по обобщенной теореме Фалеса заключаем CN 2:N 2 M=1:1 или CN 2 = N 2 M=3:2=1,5.

4) Стороны угла NАС пересекаются прямыми BM и NN 2 и по обобщенной теореме Фалеса заключаем АО: ОN=1:1,5 или АО=m ON=1,5m.

5) Стороны угла ВАN пересекаются прямыми СК и NN 1 и по обобщенной теореме Фалеса заключаем АK: KN 1 =1:1,5 или АK=n KN 1 =1,5 n .

6) KN 1 =y=1,5n.

Ответ: АК:КВ=1:3.

Замечание : Данную задачу можно было решить, используя теорему Чевы, применив ее к треугольнику АВС. По условию точки N, М, К лежат на сторонах треугольника АВС и отрезки АN, СК и ВМ пересекаются в одной точке, значит справедливо равенство: , подставим известные отношения, имеем , АК:КВ=1:3.

Задача№3 На стороне ВС треугольника АВС взята точка D такая, что ВD: DC = 2:5, а на стороне АС точка Е такая, что . В каком отношении делятся отрезки ВЕ и АD точкой К их пересечения?
Решение: Нужно найти 1) АК:КD=? 2) ВК:КЕ=?

1) Проведем прямую DD 1 параллельную прямой BE.

2) Стороны угла ВСЕ пересекаются прямыми ВЕ и DD 1 и по обобщенной теореме Фалеса заключаем CD 1:D 1 E=5:2 или CD 1 = 5z , D 1 E=2z.

3) По условию АЕ:ЕС=1:2, т.е. АЕ=х, ЕС=2х, но ЕС= CD 1 + D 1 E, значит 2у=5 z +2 z =7 z , z =

4) Стороны угла DСA пересекаются прямыми ВЕ и DD 1 и по обобщенной теореме Фалеса заключаем

5) Для определения отношения ВК:КЕ проведем прямую ЕЕ 1 и рассуждая аналогичным образом получим


Ответ: АК:КD=7:4; ВК:КЕ=6:5.
Замечание: Данную задачу можно было решить, используя теорему Менелая. Применив ее к треугольнику ВЕС. Тогда прямая DA пересекает две стороны треугольника в точках D и K, а продолжение третьей в точке A. Значит применимо равенство: , следовательно ВК:КЕ=6:5. Рассуждая аналогично относительно треугольника ADC, получим , АК:КD=7:4.
Задача №4 В ∆ ABC биссектриса AD делит сторону BC в отношении 2: 1. В каком отношении медиана CE делит эту биссектрису?

Решение: Пусть О точка пересечения биссектрисы AD и медианы СЕ. Нужно найти отношение АО:ОD.

1) Проведем прямую DD 1 параллельную прямой СE.

2) Стороны угла АВС пересекаются прямыми СЕ и DD 1 и по обобщенной теореме Фалеса заключаем ВD 1:D 1 E=2:1 или ВD 1 = 2p , D 1 E=p.

3) По условию АЕ:ЕB=1:1, т.е. АЕ=y, ЕB=y, но EB= BD 1 + D 1 E, значит у=2 p + p =3 p , p =
4) Стороны угла BAD пересекаются прямыми OЕ и DD 1 и по обобщенной теореме Фалеса заключаем .

Ответ: АО:ОD=3:1.


Задача №5 На сторонах AB и АC ∆ABC даны соответственно точки M и N такие, что выполняются следующие равенства АМ:МВ=С N : NA =1:2. В каком соотношении точка S пересечения отрезков BN и CM делит каждый из этих отрезков .

Задача №6 На медиане АМ треугольника АВС взята точка К, причем АК:КМ=1:3. Найдите отношение, в котором прямая, проходящая через точку К параллельно стороне АС, делит сторону ВС.


Решение: Пусть М 1 точка пересечения прямой, проходящая через точку К параллельно стороне АС и стороны ВС. Нужно найти отношение ВМ 1:М 1 С.

1) Стороны угла АМС пересекаются прямыми КМ 1 и АС и по обобщенной теореме Фалеса заключаем ММ 1:М 1 С=3:1 или ММ 1 = 3z, М 1 С=z

2) По условию ВМ:МС=1:1, т.е.ВМ=y, МС=y, но МС= ММ 1 + М 1 С, значит у=3 z + z =4 z ,

3) .

Ответ: ВМ 1:М 1 С =7:1.


Задача №7 Дан треугольник АВС. На продолжении стороны АС за точку С взята точка N , причем С N =АС; точка К- середина стороны АВ. В каком отношении прямая К N делит сторону ВС.

Замечание: Данную задачу можно было решить, используя теорему Менелая. Применив ее к треугольнику АВС. Тогда прямая КN пересекает две стороны треугольника в точках К и K 1 , а продолжение третьей в точке N. Значит применимо равенство: , следовательно ВК 1:К 1 С=2:1.

Задача №8

Сайты:

http://www.problems.ru

http://interneturok.ru/

ЕГЭ 2011 Математика Задача С4 Р. К.Гордин М.: МЦНМО, 2011, — 148 с

Заключение:

Решение задач и теорем на нахождение отношения длин отрезков базируется на обобщенной теореме Фалеса. Мы сформулировали метод, который позволяет, не применяя теорему Фалеса, пользоваться параллельными прямыми, переносить известные пропорции с одной стороны угла на другую сторону и, таким образом, находить место расположения нужных нам точек и сравнивать длины. Работа над рефератом помогла нам научиться решать геометрические задачи высокого уровня сложности. Мы осознали правдивость слов известного русского поэта Игоря Северянина: «Все незначительное нужно, Чтобы значительному быть…» и уверены, что на ЕГЭ мы сможем найти решение предложенным задачам, используя метод параллельных прямых.

1 Теорема о пропорциональных отрезках в треугольнике – вышеописанная теорема.

Если стороны угла, пересекают прямые параллельные линии которые одну из сторон разделяют на несколько отрезков, то и вторую сторону, прямые так же разделят на равнозначны с другой стороной отрезки.

Теорему Фалеса доказывает следующее: С 1 , С 2 , С 3 — это места где пересекаются прямые параллельные на любой стороне угла. С 2 находится посередине относительно С 1 и С 3 .. Точки D 1 , D 2 , D 3 — это места где пересекаются прямые, которые соответствуют прямым с другой стороной угла. Доказываем, что когда C 1 C 2 = C 2 C з, значит и D 1 D 2 =D 2 D 3 .
Проводим в месте D 2 прямой отрезок КР, параллельный участку C 1 C 3 . В свойствах параллелограмма C 1 C 2 =KD 2 , C 2 C 3 = D 2 P. Если C 1 C 2 =C 2 C 3 , то и KD 2 =D 2 P.

Полученные треугольные фигуры D 2 D 1 K и D 2 D 3 P равняются. И D 2 K=D 2 P по доказательству. Углы с верхней точкой D 2 равняются как вертикальные, а углы D 2 KD 1 и D 2 PD 3 равняются как внутренние накрест лежащие при параллельных C 1 D 1 и C 3 D 3 и разделяющей KP.
Так как D 1 D 2 =D 2 D 3 теорема доказана по равенству сторон треугольника

Заметка:
Если взять не стороны угла, а два прямых отрезка, доказательство будет такое же.
Любые прямые отрезки параллельные друг другу, которые пересекают две рассматриваемые нами прямые и разделяющие одну из них на одинаковые участки, тоже самое делают и со второй.

Рассмотрим несколько примеров

Первый пример

Условием задания требуется разбить прямую СD на п одинаковых отрезков.
Проводим от точки С полу-прямую с, которая не лежит на прямой СD. Отметим на ней одинаковые по величине части. СС 1 , С 1 С 2 , С 2 С 3 …..С п-1 С п. Соединяем С п с D. Проводим прямые от точек С 1 ,С 2 ,….,С п-1 которые будут параллельны относительно С п D. Прямые будут пересекать СD в местах D 1 D 2 D п-1 и разделять прямую СD на п одинаковых отрезков.

Второй пример

На стороне АВ треугольника АВС отмечена точка СК. Отрезок СК пересекает медиану АМ треугольника в точке Р, при этом АК= АР. Требуется найти отношение ВК к РМ.
Проводим через точку М прямой отрезок, параллельный СК, который пересекает АВ в точке D

По теореме Фалеса ВD=КD
По теореме пропорциональных отрезков получаем, что
РМ = КD = ВК/2, следовательно, ВК: РМ = 2:1
Ответ: ВК: РМ = 2:1

Третий пример

В треугольнике АВС, сторона ВС = 8 см. Прямая DE пересекает стороны АВ и ВС параллельно АС. И отсекает на стороне ВС отрезок ЕС = 4см. Доказать, что АD = DВ.

Так как ВС = 8 см и ЕС = 4см, то
ВЕ = ВС-ЕС, следовательно, ВЕ = 8-4 = 4(см)
По теореме Фалеса , так как АС параллельна DE и ЕС = ВЕ то, следовательно, АD = DВ. Что и требовалось доказать.

В женском журнале — онлайн, Вы найдете много интересной информации для себя. Так же есть раздел, посвященный стихам которые написал Сергей Есенин . Заходите не пожалеете!

О параллельных и секущих.

Вне русскоязычной литературы теоремой Фалеса иногда называют другую теорему планиметрии, а именно, утверждение о том , что вписанный угол , опирающийся на диаметр окружности , является прямым. Открытие этой теоремы действительно приписывается Фалесу, о чём есть свидетельство Прокла .

Формулировки

Если на одной из двух прямых отложить последовательно несколько равных отрезков и через их концы провести параллельные прямые, пересекающие вторую прямую, то они отсекут на второй прямой равные отрезки.

Более общая формулировка, также называемая теорема о пропорциональных отрезках

Параллельные прямые отсекают на секущих пропорциональные отрезки :

A 1 A 2 B 1 B 2 = A 2 A 3 B 2 B 3 = A 1 A 3 B 1 B 3 . {\displaystyle {\frac {A_{1}A_{2}}{B_{1}B_{2}}}={\frac {A_{2}A_{3}}{B_{2}B_{3}}}={\frac {A_{1}A_{3}}{B_{1}B_{3}}}.}

Замечания

  • В теореме нет ограничений на взаимное расположение секущих (она верна как для пересекающихся прямых, так и для параллельных). Также не важно, где находятся отрезки на секущих.
  • Теорема Фалеса является частным случаем теоремы о пропорциональных отрезках, поскольку равные отрезки можно считать пропорциональными отрезками с коэффициентом пропорциональности, равным 1.

Доказательство в случае секущих

Рассмотрим вариант с несвязанными парами отрезков: пусть угол пересекают прямые A A 1 | | B B 1 | | C C 1 | | D D 1 {\displaystyle AA_{1}||BB_{1}||CC_{1}||DD_{1}} и при этом A B = C D {\displaystyle AB=CD} .

Доказательство в случае параллельных прямых

Проведем прямую BC . Углы ABC и BCD равны как внутренние накрест лежащие при параллельных прямых AB и CD и секущей BC , а углы ACB и CBD равны как внутренние накрест лежащие при параллельных прямых AC и BD и секущей BC . Тогда по второму признаку равенства треугольников треугольники ABC и DCB равны. Отсюда следует, что AC = BD и AB = CD . ■

Вариации и обобщения

Обратная теорема

Если в теореме Фалеса равные отрезки начинаются от вершины (часто в школьной литературе используется такая формулировка), то обратная теорема также окажется верной. Для пересекающихся секущих она формулируется так:

В обратной теореме Фалеса важно, что равные отрезки начинаются от вершины

Таким образом (см. рис.) из того, что C B 1 C A 1 = B 1 B 2 A 1 A 2 = … {\displaystyle {\frac {CB_{1}}{CA_{1}}}={\frac {B_{1}B_{2}}{A_{1}A_{2}}}=\ldots } , следует, что A 1 B 1 | | A 2 B 2 | | … {\displaystyle A_{1}B_{1}||A_{2}B_{2}||\ldots } .

Если секущие параллельны, то необходимо требовать равенство отрезков на обеих секущих между собой, иначе данное утверждение становится неверным (контрпример — трапеция, пересекаемая линией, проходящей через середины оснований).

Этой теоремой пользуются в навигации: столкновение судов, двигающихся с постоянной скоростью, неизбежно, если сохраняется направление с одного судна на другое.

Лемма Соллертинского

Следующее утверждение, двойственно к лемме Соллертинского :

Пусть f {\displaystyle f} — проективное соответствие между точками прямой l {\displaystyle l} и прямой m {\displaystyle m} . Тогда множество прямых будет множеством касательных к некоторому коническому сечению (возможно, вырожденному).

В случае теоремы Фалеса коникой будет бесконечно удалённая точка, соответствующая направлению параллельных прямых.

Это утверждение, в свою очередь, является предельным случаем следующего утверждения:

Пусть f {\displaystyle f} — проективное преобразование коники. Тогда огибающей множества прямых X f (X) {\displaystyle Xf(X)} будет коника (возможно, вырожденная).

Тема урока

Цели урока

  • Познакомиться с новыми определениями и вспомнить некоторые уже изученные.
  • Сформулировать и доказать свойства квадрата, доказать его свойства.
  • Научиться применять свойства фигур при решении задач.
  • Развивающие – развить внимание учащихся, усидчивость, настойчивость, логическое мышление, математическую речь.
  • Воспитательные — посредством урока воспитывать внимательное отношение друг к другу, прививать умение слушать товарищей, взаимовыручке, самостоятельность.

Задачи урока

  • Проверить умение учащихся решать задачи.

План урока

  1. Историческая справка.
  2. Фалес как математик и его труды.
  3. Полезно вспомнить.

Историческая справка

  • Теорема Фалеса до сих пор используется в морской навигации в качестве правила о том, что столкновение судов, двигающихся с постоянной скоростью, неизбежно, если сохраняется курс судов друг на друга.


  • Вне русскоязычной литературы теоремой Фалеса иногда называют другую теорему планиметрии, а именно, утверждение о том, что вписанный угол, опирающийся на диаметр окружности, является прямым. Открытие этой теоремы действительно приписывается Фалесу, о чём есть свидетельство Прокла.
  • Основы геометрии Фалес постигал в Египте.

Открытия и заслуги ее автора

А известно ли вам, что Фалес Милетский был одним из семи самых известных по тем временам, мудрецом Греции. Он основал Ионийскую школу. Идею, которую продвигал Фалес в этой школе, было единство всего сущего. Мудрец считал, что есть единое начало, от которого произошли все вещи.

Огромной заслугой Фалеса Милетского является создание научной геометрии. Этот великий учений сумел с египетского искусства измерения создать дедуктивную геометрию, базой которой есть общие основания.

Кроме огромных познаний в геометрии, Фалес еще и неплохо разбирался в астрономии. Эму первому удалось предсказать полное затмение Солнца. А ведь это происходило не в современном мире, а в далеком 585 году, еще до нашей эры.

Фалес Милетский был тем человеком, который сообразил, что север можно точно определить по созвездию Малой Медведицы. Но и это не было его последним открытием, так как он сумел в точности определить продолжительность года, разбить его на триста шестьдесят пять дней, а также установил время равноденствий.

Фалес на самом деле был всесторонне развитым и мудрым человеком. Кроме того, что он славился как прекрасный математик, физик, астроном, он еще и как настоящий метеоролог, смог довольно точно предсказать урожай оливок.

Но самое примечательное то, что Фалес никогда не ограничивался в своих познаниях только научно-теоретической областью, а всегда пытался закрепить доказательства своих теорий на практике. И самое интересное, то, что великий мудрец не сосредотачивался на какой-то одной области своих познаний, его интерес имел различные направленности.

Имя Фалеса стало нарицательным для мудреца уже тогда. Его важность и значимость для Греции была так велика, как для России имя Ломоносова. Конечно, его мудрость можно толковать по-разному. Но точно можно сказать, что ему были присущи и изобретательность, и практическая смекалка, и в какой-то степени отрешенность.

Фалес Милетский был отличным математиком, философом, астрономом, любил путешествовать, был купцом и предпринимателем, занимался торговлей, а также был неплохим инженером, дипломатом, провидцем и активно участвовал в политической жизни.

Он даже умудрился с помощью посоха и тени определить высоту пирамиды. А было это так. В один погожий солнечный день Фалес поставил свой посох на границе, где заканчивалась тень от пирамиды. Далее он дождался, когда длинна от тени его посоха сравнялась с его высотой, и замерил длину тени пирамиды. Вот так, казалось бы просто Фалес определил высоту пирамиды и доказал, что длина одной тени имеет отношение к длине другой тени, также, как и высота пирамиды относится к высоте посоха. Чем и поразил самого фараона Амасиса.

Благодаря Фалесу все известные в то время знания были переведены в область научного интереса. Он смог донести результаты до уровня, пригодного для научного потребления, выделив определенный комплекс понятий. И возможно с помощью Фалеса началось последующее развитие античной философии.

Теорема Фалеса играет одну важных ролей в математике. Она была известна не только в Древнем Египте и Вавилоне, но и в других странах и являлась почвой для развития математики. Да и в повседневной жизни, при строительстве зданий, сооружений, дорог и т.д., без теоремы Фалеса не обойтись.

Теорема Фалеса в культуре

Теорема Фалеса прославилась не только в математике, но ее приобщили еще и к культуре. Однажды аргентинская музыкальная группа Les Luthiers (исп.) на суд зрителей представила песню, которую посвятила известной теореме. Участники Les Luthiers в своем видеоклипе специально для этой песни предоставили доказательства для прямой теоремы для пропорциональных отрезков.

Вопросы

  1. Какие прямые называются параллельными?
  2. Где практически применяется теорема Фалеса?
  3. О чем гласит теорема Фалеса?

Список использованных источников

  1. Энциклопедия для детей. Т.11. Математика/Глав.ред.М.Д.Аксенова.-м.:Аванта+,2001.
  2. «Единый государственный экзамен 2006. Математика. Учебно-тренировочные материалы для подготовки учащихся/ Рособрнадзор, ИСОП – М.: Интеллект-Центр, 2006»
  3. Л. С. Атанасян, В. Ф. Бутузов, С. Б. Кадомцев, Э. Г. Позняк, И. И. Юдина «Геометрия, 7 – 9: учебник для общеобразовательных учреждений»
Предмети > Математика > Математика 8 класс

Теорема Фалеса — это… Что такое Теорема Фалеса?

Эта теорема о параллельных прямых. Об угле, опирающемся на диаметр, см. другую теорему.

Теорема Фалеса — одна из теорем планиметрии.

Если на одной из двух прямых отложить последовательно несколько равных отрезков и через их концы провести параллельные прямые, пересекающие вторую прямую, то они отсекут на второй прямой равные между собой отрезки.

В теореме нет ограничений на взаимное расположение секущих (она верна как для пересекающихся прямых, так и для параллельных). Также неважно, где находятся отрезки на секущих.

Доказательство в случае секущих  

Доказательство в случае параллельных прямых  

Проведем прямую BC. Углы ABC и BCD равны как внутренние накрест лежащие при параллельных прямых AB и CD и секущей BC, а углы ACB и CBD равны как внутренние накрест лежащие при параллельных прямых AC и BD и секущей BC. Тогда по первому признаку равенства треугольников треугольники ABC и DCB равны. Отсюда следует, что AC = BD и AB = CD. ■

Также существует обобщённая теорема Фалеса:

Параллельные прямые отсекают на секущих пропорциональные отрезки:

Теорема Фалеса является частным случаем обобщённой теоремы Фалеса, поскольку равные отрезки можно считать пропорциональными отрезками с коэффициентом пропорциональности, равным 1.

Обратная теорема

Если в теореме Фалеса равные отрезки начинаются от вершины (часто в школьной литературе используется такая формулировка), то обратная теорема также окажется верной. Для пересекающихся секущих она формулируется так:

Если прямые, пересекающие стороны угла, отсекают на одной и на другой стороне угла равные (или пропорциональные) между собой отрезки, начиная от вершины, то такие прямые параллельны.

В обратной теореме Фалеса важно, что равные отрезки начинаются от вершины

Таким образом (см. рис.) из того, что следует, что прямые .

Если секущие параллельны, то необходимо требовать равенство отрезков на обеих секущих между собой, иначе данное утверждение становится неверным (контрпример — трапеция, пересекаемая линией, проходящей через середины оснований).

Вариации и обобщения

Следующее утверждение, двойственно к лемме Соллертинского:


В случае теоремы Фалеса коникой будет бесконечно удалённая точка, соответствующая направлению параллельных прямых.

Это утверждение, в свою очередь, является предельным случаем следующего утверждения:

Пусть  — проективное преобразование коники. Тогда огибающей множества прямых будет коника (возможно, вырожденная).

Теорема Фалеса в культуре

Аргентинская музыкальная группа Les Luthiers (исп.) представила песню, посвящённую теореме. В видеоклипе для этой песни[1] приводится доказательство для прямой теоремы для пропорциональных отрезков.

Интересные факты

  • Теорема Фалеса до сих пор используется в морской навигации в качестве правила о том, что столкновение судов, двигающихся с постоянной скоростью, неизбежно, если сохраняется курс судов друг на друга.
  • Вне русскоязычной литературы теоремой Фалеса иногда называют другую теорему планиметрии, а именно, утверждение о том, что вписанный угол, опирающийся на диаметр окружности, является прямым. Открытие этой теоремы действительно приписывается Фалесу, о чём есть свидетельство Прокла.

Литература

  • Атанасян С.Л. Геометрия 7-9. — Изд. 3-е. — М.: Просвещение, 1992.

Примечания

Теорема Фалеса.

Пропорциональные Краткосрочный план

Раздел:

Школа:

Дата:

ФИО учителя:

КЛАСС: 8

Количество присутствующих:

отсутствующие:

Тема урока

Теорема Фалеса

Цели обучения

8.3.1.6Знает и применяет теорему Фалеса

8.3.1.7 Знает и применяет теорему о пропорциональных отрезках (обобщенная теорема Фалеса)

Цели урока

Знает теорему Фалеса

Составляетуравнение основываясь на теореме Фалеса

Доказывает обобщенную теорему Фалеса

Критерии оценвания

Среди показанных чертежей определяет случаи, в которых можно применить теорему Фалеса

Решает задачи применяя теорему Фалеса

Доказывает что средняя линия треугольника равна половине основания (некоторые)

Составляет пропорцию основываясь на обобщенную теорему Фалеса

Языковые цели

Фалес теоремасы – теорема Фалеса —  Thales’ theorem

Теңбөліктер – равные части — equalparts

Параллель түзулер – Параллельные прямые — Parallellines

Привитие ценностей

Ответственность, уважение к друг-другу.

Межпредметныесвязи

Раздел закладывает основу для дальнейшего изучения геометрий(пример: в 9-классе будет применять при доказательстве подобия треугольников).

Навыки использования ИКТ

GeogebraИМС, Youtube, интерактивная доска

Предварительные знания

Параллельные прямые, отрезок, угол, равенство треугольников,теорема о катете, противолежащем углу в 300

План

Запланиров-анные этапы урока

Запланированные действия

 

Ресурсы

Начало урока

6 мин

Приветствие

Учитель знакомит с целями и критериями оценивания урока

Вопрос-ответ:

Учитель задает вопросы, направленные на повторение: параллельные прямые, отрезок, угол, равенство треугольников, теорема о катете, противолежащем углу в 300

 

1-3 слайд

Середина урока

5 мин

 

 

7 мин

 

 

 

 

 

 

7 мин

 

 

 

 

 

 

 

15 мин

 

 

 

 

 

20 мин

 

 

 

 

 

 

 

15 мин

 

 

Учитель раздает учащимся треугольники, и делит учащихся на 3 группы в зависимости от вида треугольника (вид треугольника относительно угла)

 

1 задача:Раздать каждой группе листы с теоремой Фалеса и ее доказательством (дидактический материал). Дать учащимся 7 минут на ознакомление с теоремой. (Можно включить видео вместо таймера)

 

Что бы оценить насколько было освоено теорема Фалеса рассмотреть следующую задачу.

2 задача: Нужно определить в каких из представленных рисунков можно применить теорему Фалеса.

Каждый записывает в тетрадь номера рисунков в которых, по их мнению, можно применить теорему Фалеса (1 мин). После обсуждения каждая группа приводит свой ответ (2 мин). В конце, класс определяет общий ответ (2 мин).

 

Следующую задачу нужно решить с помощью Теоремы Фалеса.

3 задача:Нужно группой решить задачу (раздаточный материал 2, 10 мин).

Основную задачу все должны выполнить.

Учитель разбирает задачу и доказательство

 

Доказательство обобщенной теоремы Фалеса

4 задача: Учитель раздает схему (раздаточный материал 3), по которой каждая группа приводит свой вариант доказательства теоремы. Можно рассмотретьнесколько вариантов доказательства.

 

Задача на применение обобщенной теоремы Фалеса

5 задача:

Сначала каждый решает задачу самостоятельно (7 мин), затем задача обсуждается внутри группы,и каждая группа приводит свой вариант решения (4 мин), в конце класс путем обсуждения приходит к общему решению (4 мин).

раздаточный материал1

 

 

дидактический материал

Фалес.Биография (7 мин)

 

4 слайд

 

 

 

 

 

 

 

раздаточный материал2

5 слайд –доказательство

 

 

 

раздаточный материал3

 

 

 

 

 

 

6 слайд

Конец урока

 

 

5 мин

Домашнее задание:

Дать на дом дополнительную задачу с раздаточного

материала 2.

 

Составить две задачи на применение теоремы Фалеса:

 

1. Задача в виде чертежа (на теорему Фалеса)

2. Текстовая задача (На обобщённую теорему Фалеса)

 

Рефлексия: Заполнить таблицу

Понятия, рассмотренные на уроке

Я понял

Я не понял

 

 

 

 

 

 

 

 

 

 

Дифференциация — как вы планируете оказывать больше поддержки? Как вы планируете давать задания более способным учащимся?

Оценивание — как вы планируете проверить знания учащихся?

Междисциплинарные связи

Безопасность жизнедеятельности

ИКТ связи

Связи с ценностями

 

 

 

 

 

 

 

 

 

 

 

 

 

Размышление

Были ли цели урока / цели обучения реалистичными?

Что учащиеся узнали сегодня?

Какова была атмосфера обучения?

Была ли эффективна запланированная дифференциация?

Удалось ли придерживаться запланированного времени? Какие изменения в плане я бы сделал(-а) и почему?

Используйте графу ниже, чтобы проанализировать Ваш урок. Ответьте на самые актуальные вопросы из графы слева о проведенном уроке.

 

 

 

 

 

 

 

 

 

 

 

 

 

Сводная оценка

Какие два аспекта прошли очень хорошо (рассмотрите преподавание и обучение)?

 

 

 

 

 

 

Какие два аспекта улучшили бы урок (рассмотрите преподавание и обучение)?

 

 

 

 

 

 

Что я узнал о классе или об отдельных учащихся на данном уроке, что я учту на следующем уроке?

 

 

 

 

 

 

«Применение теоремы Фалеса для решения практических задач» ( 8 класс)

Пояснительная записка.

Урок геометрии в 8 классе по теме: «Применение теоремы Фалеса для решения практических задач» является 4 уроком в теме «Параллелограмм и трапеция», рассчитанной на 6 часов. Урок практического применения полученных знаний.

Программа общеобразовательных учреждений Геометрия 7-9 классы составитель Т.А.Бурмистрова, 68 часов за год , 2 часа в неделю. Автор учебника — Л.С.Атанасян.

Характеристика класса.

Репродуктивный;

Проблемный;

Частично-поисковый;

Исследовательский

Предпочитают проблемный и поисковый методы обучения.

6. Отношение к различным формам учебной деятельности, в том числе и нестандартных

Принимают различные формы обучения, предпочтение отдают групповой работе, либо соревнование в командах.

Этапы урока.

Домашнее задание:

— выучить теорему Фалеса;

— разделить отрезок на 9 равных частей.

3.Подготовка учащихся к усвоению актуализация опорных знаний.

5

Объяснительно-иллюстративный.

Проверка домашнего задания. Задача, решённая дома встречается при доказательстве теоремы Фалеса, как элемент доказательства.

4.Мотивация учебной деятельности.

5

Проблемная ситуация.

Целеполагание.

Учащимся предлагается разделить отрезок на 5 равных частей. Учащиеся предлагают свои решения, с помощью учителя идёт проверка версий на их состоятельность.

Встаёт проблема деления отрезка на равные части.

5.Изучение нового материала.

10

Использование презентации.

Фронтальная форма работы.

При участии учащихся разбирается доказательство теоремы. Здесь участвует, как элемент доказательства домашняя задача, поэтому один из учащихся выходит к доске и проводит доказательство части теоремы.

6.Применение теории при решении задач.

6

Анализ, сравнение. Воспроизведение.

Использование презентации.

Целеполагание.

Вопрос учителя: Каким образом можно применить теорему, чтобы разделить отрезок на равные части? Если ученик выдвигает устно правильное решение, то учитель показывает с помощью проектора слайды пошагового выполнения действий.

Вопрос учителя: Чьё имя носит эта теорема? Формулируется тема урока.

7.Первичное закрепление нового материала.

7

Письменная работа в тетрадях.

Задание учащимся: Выполнить деление отрезка на 5 равных частей. Учащиеся выполняют построения, потом самостоятельно составляют алгоритм деления отрезка на равные части. Обсуждаем. После обсуждения в справочную тетрадь записываем алгоритм.

8.Первичная проверка усвоения знаний.

9

Работа в группах.

Задания в группах: Разделить ленту (тесьму, верёвку, шнур, деревянную рейку (для мальчиков)) на 3 равные части.

Отчёт групп о проделанной работе.

9.Подведение итогов урока. Рефлексия.

2

Анализ. Беседа

Продолжить предложения:
— На уроке я узнал…
— На уроке научился…

Урок, конспект которого я предлагаю – это урок практического применения полученных знаний. Целями данного урока являлись: развивать у учащихся умение выделять главное, применять раннее полученные знания, сравнивать, анализировать; формирование осознанного отношения к жизненным ситуациям, понимания собственной значимости, аккуратности, умения работать в группе; доказать теорему Фалеса; применить теорему Фалеса при решении задач.

Урок построен на принципах развивающего обучения, что позволило активно задействовать творческую и исследовательскую деятельность.

Особенностями урока является:

  1. Интенсивная самостоятельная деятельность учащихся, связанная с эмоциональным переживанием, которая сопровождается эффектом неожиданности задания; помощью и поощрением со стороны учителя.

  2. Создание педагогических ситуаций общения на уроке, позволяющих каждому ученику проявлять инициативу, самостоятельность.

  3. Создание обстановки для естественного самовыражения ученика.

Всё это позволило учащимся активно включиться в работу с самого начала урока. Успешность и активность работы была достигнута за счет системы заданий, поступающих мелкими порциями. Домашняя задача, разобранная у доски в начале урока, дала возможность учащимся самостоятельно доказать один из этапов доказательства теоремы Фалеса, что создало ситуацию успеха для «слабых» учеников. Наибольший интерес вызвала групповая работа, позволившая практически применить полученные знания. С поставленным заданием учащиеся успешно справились, а значит, цели урока были достигнуты.

Сценарий урока.

Тема урока: «Применение теоремы Фалеса для решения практических задач».

8 класс.

Цели урока:

Развивающая: развивать у учащихся ключевые компетенции — умение выделять главное, применять раннее полученные знания, сравнивать, анализировать;

Воспитывающая: продолжить формирование мировоззрения обучающихся, осознанного отношения к жизненным ситуациям, понимания собственной значимости, аккуратности, умения работать в группе;

Образовательная: сформировать предметные компетенции, доказать теорему Фалеса; применить теорему Фалеса при решении задач.

Ход урока.

  1. Организационный момент.

  2. Запись домашнего задания: — выучить доказательствотеоремы Фалеса;

— разделить отрезок на 9 равных частей.

  1. Проверка домашнего задания.

На доске заготовлен рисунок. Один учащийся выходит к доске и проводит доказательство.

  1. Мотивация.

На доске заготовлен отрезок. Учитель формулирует задание:

-разделите отрезок на 4 равные части;

-разделите отрезок на 5 равных частей.

Ясно, что встаёт проблема деления отрезка на равные части. С этой проблемой столкнулись учёные не сейчас, не в этом столетии, а на много веков раннее. И чтобы нам сегодня справиться с возникшей задачей докажем одну из важнейших теорем геометрии.

  1. Изучение нового материала, доказательство теоремы.

Теорема: Если на одной из двух прямых отложить последовательно несколько равных отрезков и через их концы провести параллельные прямые, пересекающие вторую прямую, то они отсекут на второй прямой равные между собой отрезки.

При участии учащихся разбирается доказательство теоремы. Здесь участвует, как элемент доказательства домашняя задача.

а)

  1. Применение теоремы для деления отрезка на равные части.

Вопрос учителя: Как с помощью этой теоремы можно разделить отрезок на равные части?

Учащиеся высказывают своё мнение. Если произносятся верные действия, то с помощью проектора учитель показывает этапы деления отрезка на равные части.

Итак, решение проблемы есть, отрезок разделён на равные части, но кто же был «автором» этой теоремы, чьё имя носит теорема. Здесь учитель обращается к теме урока: «Теорема Фалеса». Фалес — древнегреческий учёный, живший в 624-547г.г. до н.э.

Сопутствующий слайд проектора:

  1. Работа в тетрадях.

Задание: Разделите отрезок на 5 равных частей с помощью циркуля и линейки.

Учащиеся выполняют работу, при этом учитель снова показывает слайды по делению отрезка на равные части. Учитель имеет возможность помочь учащимся, которым сложно справиться с заданием.

При участии учащихся записываем в тетрадь алгоритм деления отрезка на равные части.

Алгоритм:

1.Провести луч из конца отрезка.

2.На луче с помощью циркуля отметить равные отрезки.

3.Конец последнего отрезка соединить со свободным концом данного отрезка.

4. Через концы равных отрезков провести прямые параллельные, полученной прямой, до

пересечения с отрезком.

5.Отрезок разделился на равные части.

  1. Практическое применение теоремы Фалеса.

Класс делится на группы по 4-5 человек.

Задание для группы: Разделите верёвку (деревянный брусок, ленту, тесьму, шнур) на 5 равных частей.

  1. Подведение итогов урока. Рефлексия.
    Продолжить предложения:
    — На уроке я узнал…
    — На уроке научился…

Что такое Аксиома и Теорема? Определение, примеры, доказательства.

Понятие аксиомы

Аксиома — это правило, которое считают верным и которое не нужно доказывать. В переводе с греческого «аксиома» значит принятое положение — то есть взяли и договорились, что это истина, с которой не поспоришь.

Аксиоматический метод — это подход к получению знаний, при котором сначала разрабатывают аксиомы, а потом с их помощью формулируют новые теории.

Синоним аксиомы — постулат. Антоним — гипотеза.

Основные аксиомы евклидовой геометрии

 
  1. Через любые две точки проходит единственная прямая.

  2. Каждая точка на прямой разбивает эту прямую на две части так, что точки из разных частей лежат по разные стороны от данной точки. А точки из одной части лежат по одну сторону от данной точки.

  3. На любом луче от его начала можно отложить только один отрезок, равный данному.

  4. Отрезки, полученные сложением или вычитанием соответственно равных отрезков — равны.

  5. Каждая прямая на плоскости разбивает эту плоскость на две полуплоскости. При этом если две точки принадлежат разным частям, то отрезок, который соединяет эти две точки, пересекается с прямой. Если две точки принадлежат одной части, то отрезок, соединяющий эти точки, не пересекается с прямой.

  6. От любого луча на плоскости в заданную сторону можно отложить только один угол, который равен данному. Все развернутые углы равны.

  7. Углы равны, если они получились путем сложения или вычитания соответственно равных углов.

Учить наизусть эти аксиомы не обязательно. Главное — помнить о них и держать под рукой, чтобы при доказательстве теоремы сослаться на одну из них.

А теперь давайте рассмотрим несколько аксиом из геометрии за 7 и 8 класс.

Самая известная аксиома Евклида — аксиома о параллельных прямых. Звучит она так:

Это значит, что если дана прямая и любая точка, которая не лежит на этой прямой, то через неё можно провести только одну единственную прямую, которая будет параллельна этой первой данной прямой.


У этой аксиомы два следствия:

  • прямая, которая пересекает одну параллельную прямую, обязательно пересекает и другую;
  • если две прямые параллельны третьей, то между собой они также параллельны.

Аксиома Архимеда заключается в том, что, если отложить достаточное число раз меньший из двух отрезков, то можно покрыть больший из них. Звучит так:

Если на прямой есть меньший отрезок А и больший отрезок B, то, можно сложить А достаточное количество раз, чтобы покрыть B.

На картинке можно увидеть, как это выглядит:


Из этого следует, что не существует бесконечно малых и бесконечно больших величин. В качестве математической формулы аксиому можно записать так: А + А + … + А = А * n > В, где n — это натуральное число.

Понятие теоремы

Что такое аксиома мы уже поняли, теперь узнаем определение теоремы.

Теорема — логическое следствие аксиом. Это утверждение, которое основано на аксиомах и общепринятых утверждениях, которые были доказаны ранее, и доказывается на их основе.

Состав теоремы: условие и заключение или следствие.

Среди теорем выделяют такие, которые сами по себе не используются в решениях задач. Но их используют для доказательства других теорем.

Лемма — это вспомогательная теорема, с помощью которой доказываются другие теоремы. Пример леммы: если одна из двух параллельных прямых пересекает плоскость, то и вторая прямая тоже пересекает эту плоскость.

Следствие — утверждение, которое выводится из аксиомы или теоремы. Следствие, как и теорему, необходимо доказывать.

Примеры следствий из аксиомы о параллельности прямых:

  • если прямая пересекает одну из двух параллельных прямых, то она пересекает и другую;
  • если две прямые параллельны третьей прямой, то они параллельны.

Доказательство теоремы — это процесс обоснования истинности утверждения.

Каждая доказанная теорема служит основанием доказательства для следующей теоремы. Именно поэтому так важно изучать геометрию последовательно, переходя от аксиом к теоремам.

Способы доказательства геометрических теорем

  • Синтетический или синтез — метод, при котором данное предложение выступает, как необходимое следствие другого, уже доказанного.
  • Аналитический или анализ — обратный синтезу способ. Рассуждения всегда начинаются с доказываемой теоремы и закачиваются другой известной истиной.

Часть аналитического способа — доказательство от противного, когда для доказательства данного предложения убеждают в невозможности предположения противоположного.

Приемы для доказательства в геометрии:

  • Способ наложения — когда одну геометрическую величину накладывают на другую. Этим способом убеждаются в равенстве или неравенстве геометрических протяжений в зависимости от того, совмещаются они или нет при наложении.
  • Способ пропорциональности — применение свойств пропорций. Этот способ пригодится для доказательства теорем про подобные фигуры и пропорциональные отрезки.
  • Способ пределов — когда вместо данной величины берут свойства другой, близкой к ней. А потом перекладывают эти выводы на исходные данные.

Обратная теорема — это такой перевертыш: в ней условие исходной теоремы дано заключением, а заключение — условием.

Прямая и обратная теорема взаимно-обратные. Например:

  • прямая теорема: в треугольнике против равных сторон лежат равные углы.
  • обратная теорема: в треугольнике против равных углов лежат равные стороны.

В первой теореме данное условие — это равенство сторон треугольника, а заключение — равенство противолежащих углов. А во второй всё наоборот.

Противоположная теорема — это утверждение, в котором из отрицания условия вытекает отрицание заключения.

Вот, как выглядит взаимное отношение теорем на примере:

  • Прямая: если при пересечении двух прямых третьей соответственные углы равны, то данные прямые параллельны.
  • Обратная: если две прямые параллельны, то при пересечении их третьей, соответственные углы равны.
  • Противоположная: если при пересечении двух прямых третьей соответственные углы не равны, прямые не параллельны.
  • Обратная противоположной: если прямые не параллельны, соответственные углы не равны.

В геометрическом изложении достаточно доказать только две теоремы, тогда остальные справедливы без доказательства.

Для тех, кто хочет связать свою жизнь с точными науками, Skysmart предлагает курс подготовки к ЕГЭ по математике (профиль).

Теоремы без доказательств

Теорема Пифагора: квадрат гипотенузы равен сумме квадратов катетов.

Доказательств может быть несколько. Одно из них звучит так: если построить квадраты на сторонах прямоугольного треугольника, то площадь большего из них равна сумме площадей меньших квадратов. На картинке понятно, как это работает:


Теорема косинусов: квадрат одной стороны треугольника равен сумме квадратов двух других сторон минус удвоенное произведение этих сторон на косинус угла между ними. В виде формулы это выглядит так:


где a, b и c — стороны плоского треугольника,

α — угол, противолежащий стороне а.


Следствия из теоремы косинусов:


  • при b² + c² – a² > 0 угол α будет острым;
  • при b² + c² – a² = 0 угол α будет прямым, что соответствуем теореме Пифагора;
  • при b² + c² – a² < 0 угол α будет тупым.

Теорема синусов: стороны треугольника пропорциональны синусам противолежащих углов.

Формула:


где a, b, c — стороны треугольника,

α, β, γ — углы, противоположные сторонам a, b, c соответственно.


Понятия свойств и признаков

У нас есть список аксиом и мы уже знаем, что такое теорема и как ее доказывать. Есть два типа утверждений среди теорем, которые часто встречаются при изучении новых фигур: свойства и признаки.

Свойства и признаки — понятия из обычной жизни, которые мы часто используем.

Свойство — такое утверждение, которое должно выполняться для данного типа объектов. У ноутбука есть клавиатура — это свойство есть у каждого ноутбука. А у электронной книги такого свойства нет.

Примеры геометрических свойств мы уже знаем: у квадрата все стороны равны. Это верно для любого квадрата, поэтому это — свойство.

Такое свойство можно встретить у другого четырехугольника. И клавиатура может быть на других устройствах, помимо ноутбука. Из этого следует, что свойства не обязательно должны быть уникальными.

Признак — это то, по чему мы однозначно распознаем объект.

Звезды в темном небе — признак того, что сейчас ночь. Если человек ходит с открытым зонтом — это признак того, что сейчас идет дождь. При этом ночью не обязательно должны быть видны звезды, иногда может быть облачно. Значит это не свойство ночи.

А теперь вернемся к геометрии и рассмотрим четырехугольник ABCD, в котором AB = BD = 10 см.

Является ли равенство диагоналей признаком прямоугольника? У такого четырехугольника, где AB = BD, диагонали равны, но он не является прямоугольником. Это свойство, но не его признак.


Но если в четырехугольнике противоположные стороны параллельны AB || DC и AD || BC и диагонали равны AB = BD, то это уже верный признак прямоугольника. Смотрите рисунок:


Иногда свойство и признак могут быть эквивалентны. Лужи — это верный признак дождя. У других природных явлений не бывает луж. Но если приходит дождь, то лужи на асфальте точно будут. Значит, лужи — это не только признак, но и свойство дождя.

Такие утверждения называют необходимым и достаточным признаком.

Теорема Фалеса — математический путь

Есть несколько теорем, которые приписываются Фалесу Милетскому , мы сосредоточимся особенно на двух из них:

Теорема Фалеса

Теорема Фалеса — это частный случай теоремы о вписанном угле, она относится к прямоугольным треугольникам, вписанным в окружность.

Теорема Фалеса утверждает, что если A , B и C являются разными точками на окружности с центром O ( центр описанной окружности ), где прямая AC является диаметром, то треугольник Δ ABC имеет прямой угол (90) в точке B .Таким образом, Δ ABC — прямоугольный треугольник.

Другими словами, диаметр круга всегда образует прямой угол с любой точкой на окружности.

Доказательство теоремы Фалеса

Если мы соединим центр описанной окружности O с точкой B , мы создадим два треугольника Δ ABO и Δ OBC , которые являются равнобедренными треугольниками, потому что все радиусы r равны ( OA , OB и OC равны).И, согласно теореме о базовом угле, их базовые углы равны. Обозначим базовые углы Δ ABO ‘α’ и Δ ABO ‘β’.

Поскольку это равнобедренные треугольники, каждый из них имеет два равных угла: α и β (см. Рисунок выше).

Как и в любом треугольнике, внутренние углы треугольника Δ ABC в сумме составляют 180 °:

Делим равенство на 2:

Поскольку α + β — это угол Δ ABC в точке B , Теорема Фалеса доказана .

Теорема о перехвате

Теорема о перехвате гласит, что если две пересекающиеся прямые разрезаются параллельными линиями, отрезки отрезков, отрезанные параллельными прямыми от одной из прямых, пропорциональны соответствующим отрезкам отрезка от другой прямой.

Если любые две линии (на изображении: m, и n ) разрезаны серией параллельных линий (на изображении: r , s и t ), образующиеся отрезки в одной из них пропорциональны соответствующим отрезкам, образованным в другой линии.

Применяя теорему о перехвате , верно, что:

Где r — отношение.

Теорема о перехвате связана с подобием. Фактически, это эквивалентно понятию подобных треугольников. Применяя теорему о перехвате к треугольникам, мы можем утверждать, что если в данном треугольнике мы проведем линию, параллельную одной из его трех сторон, новый сгенерированный треугольник будет похож на первый. То есть мы создадим два равных треугольника, которые будут в Thales в позиции .

См. Рисунок выше.

В Δ ABC рисуем отрезок линии A’C ’, параллельный стороне AC . Появляется новый Δ A’BC ’, похожий на первый. У них три равных угла, а их стороны пропорциональны.

Согласно теореме о перехвате верно, что:

Это соотношение сохраняется между двумя сторонами одного и того же треугольника, а также между соответствующими сторонами другого:

Знаете ли вы , что Фалес Милетский (родился на Ионическом острове Милет в 7 веке до нашей эры) считался одним из семи мудрецов Греции? Он преуспел в философии, астрономии, геометрии, инженерии и … даже политике).

Под влиянием египетских и вавилонских знаний было сказано (поддержанное, среди прочего, Плутархом), что на основе своей первой теоремы и путем измерения теней он определил высоту пирамид Гизы.

Основная теорема о пропорциональности | Теорема Фалеса | Заявление и доказательство

Основная теорема пропорциональности была предложена известным греческим математиком Фалесом, поэтому ее также называют теоремой Фалеса . По словам известного математика, для любых двух равноугольных треугольников отношение любых двух соответствующих сторон данных треугольников всегда одинаково.На основе этой концепции была предложена основная теорема пропорциональности (BPT). Он показывает отношения между сторонами любых двух равносторонних треугольников.

Понятие теоремы Фалеса было введено в подобных треугольниках. Если данные два треугольника похожи друг на друга, то

  • Соответствующие углы обоих треугольников равны
  • Соответствующие стороны обоих треугольников пропорциональны друг другу

Таким образом, теорема также помогает нам лучше понять концепцию подобных треугольников.Теперь давайте попробуем понять основную теорему пропорциональности.

Формулировка основной теоремы о пропорциональности

Основная теорема пропорциональности, также известная как теорема Фалеса, гласит, что «линия, проведенная параллельно одной стороне треугольника и пересекающая две другие стороны, делит две другие стороны в равной пропорции». Например, на данном рисунке линия DE проведена параллельно стороне BC, так что она соединяет две другие стороны, AB и AC.Согласно основной теореме пропорциональности, можно заключить, что AD / DB = AE / EC.

Доказательство основной теоремы о пропорциональности

Давайте теперь попробуем доказать утверждение основной теоремы пропорциональности (BPT).

Заявление: Линия, проведенная параллельно одной стороне треугольника и пересекающая две другие стороны, делит две другие стороны в равной пропорции.

Дано: Рассмотрим треугольник ΔABC, как показано на данном рисунке.В этом треугольнике проведем прямую DE, параллельную стороне BC треугольника ABC и пересекающую стороны AB и AC в точках D и E соответственно.

Конструкция: На приведенной выше диаграмме создайте воображаемые линии, где вы можете соединить C с D и B с E. Нарисуйте перпендикуляр DP, перпендикулярный AE, и EQ, перпендикулярный AD.

Проба:

Рассмотрим треугольники ADE и BDE. Оба этих треугольника находятся на одном основании AB и имеют одинаковую высоту EQ.

(Область ADE) / (Область BDE) = (1/2 × AD × EQ) / (1/2 × BD × EQ)

(Площадь ADE) / (Площадь BDE) = AD / BD

Теперь рассмотрим треугольники CDE и ADE. Оба этих треугольника находятся на одном основании AC и имеют одинаковую высоту DP.

(Площадь ADE) / (Площадь CDE) = (1/2 × AE × DP) / (1/2 × CE × DP)

(Площадь ADE) / (Площадь CDE) = AE / CE

Оба треугольника BDE и CDE находятся между одним и тем же набором параллельных прямых.

Площадь треугольника BDE = Площадь треугольника CDE

Применяя это, мы имеем (Площадь треугольника ADE) / (Площадь треугольника BDE) = (Площадь треугольника ADE) / (Площадь треугольника CDE)

AD / BD = AE / CE

Следствие:

Приведенное выше доказательство также полезно для доказательства другой важной теоремы, называемой теоремой о средней точке.Теорема о средней точке утверждает, что отрезок прямой, проведенный параллельно одной стороне треугольника, и половина этой стороны делит две другие стороны в средних точках.

Заключение:

Таким образом, мы доказываем основную теорему о пропорциональности. Следовательно, прямая DE, проведенная параллельно стороне BC треугольника ABC, делит две другие стороны AB, AC в равной пропорции. Также верна обратная теорема о средней точке BPT. В нем говорится, что линия, проведенная через середину стороны треугольника, параллельной другой стороне, делит третью сторону треугольника пополам.

Обращение к основной теореме о пропорциональности

Согласно обратной теореме пропорциональности: «Если отрезок прямой рассекает две стороны треугольника в равной пропорции, то он параллелен третьей стороне».

Дано:

ABC представляет собой треугольник, и прямая DE разрезает стороны AB и AC в равной пропорции. AD / BD = AE / CE

Проба:

Считайте, что DE не параллельна BC.Поэтому проведем другую линию DF, параллельную BC. Применяя основную теорему о пропорциональности, получаем: AD / BD = AF / CF. Но уже дано, что: AD / BD = AE / CE. Наблюдая равные левые части двух приведенных выше утверждений, мы заключаем следующее утверждение: AE / CE = AF / CF. Добавьте 1 с обеих сторон этого утверждения.

(AE / CE) + 1 = (AF / CF) + 1

(AE + CE) / CE = (AF + CF) / CF

AC / CE = AC / CF

∴ CE = CF

Для приведенного выше утверждения точки E и F — это одни и те же точки, и они совпадают.Следовательно, прямая DE параллельна BC, и это доказывает обратное основной теореме пропорциональности.

Важные примечания

  • Основная теорема о пропорциональности — линия, проведенная параллельно одной стороне треугольника и пересекающая две другие стороны, делит две другие стороны в равной пропорции.
  • Обратное к основной теореме пропорциональности — линия, проведенная, чтобы разрезать две стороны треугольника в равной пропорции, параллельна третьей стороне.
  • Теорема о средней точке — линия, проведенная параллельно одной стороне треугольника и половине этой стороны, разделяет две другие стороны в его средней точке.

Сложные вопросы

  • Диагонали четырехугольника PQRS пересекаются в точке O, так что PO / QO = RO / SO. Докажите, что PQRS — это трапеция.


Часто задаваемые вопросы по основной теореме пропорциональности

Что такое теорема Фалеса?

Теорема Фалеса, которую также называют основной теоремой пропорциональности, утверждает, что линия, проведенная параллельно одной стороне треугольника и пересекающая две другие стороны, делит эти две стороны в равной пропорции.

Каковы приложения основной теоремы о пропорциональности?

Основная теорема пропорциональности помогает найти длины, на которых две стороны треугольника разделены линией, проведенной параллельно третьей стороне. Кроме того, у него есть приложения для нахождения взаимосвязи между двумя равноугольными треугольниками.

Что такое история теоремы Фалеса?

Теорема Фалеса была предложена Фалесом, греческим математиком и философом около 625 г. до н.э.Теперь это называется основной теоремой пропорциональности, и она помогает найти соотношение между сторонами двух равноугольных треугольников.

Какова формула основной теоремы о пропорциональности?

Базовая формула теоремы пропорциональности для треугольника ABC с точкой D на AB, точкой E на AC и DE // BC выглядит следующим образом:

AD / DB = AE / EC

Что вы имеете в виду под основной теоремой о пропорциональности?

Основная теорема пропорциональности гласит, что если линия проводится параллельно одной стороне треугольника и пересекает две другие стороны, то она делит две другие стороны в равной пропорции.

Как доказать основную теорему о пропорциональности вырезанием из бумаги?

Чтобы показать основную теорему пропорциональности, вырежьте из цветной бумаги треугольник и отметьте его вершины как ABC. Поместите его на бумагу в линейку так, чтобы одна сторона ВС совпадала с линией на бумаге в линейку. Теперь отметьте точки D на AB и E на AC так, чтобы DE была параллельна стороне BC. Теперь измерьте длины AD, BD, AE и CE и проверьте, пропорциональны ли они.

AD / DB = AE / EC

Как решить теорему Фалеса?

Теорема Фалеса аналогична основной теореме пропорциональности.Чтобы решить эту проблему, мы должны доказать, что линия, проведенная параллельно одной стороне треугольника, делит две другие стороны в равной пропорции.

Основная теорема пропорциональности

Covid-19 привел мир к феноменальному переходу.

За электронным обучением будущее уже сегодня.

Оставайтесь дома, оставайтесь в безопасности и продолжайте учиться !!!

Основная теорема пропорциональности (теорема Фалеса): Если линия проводится параллельно одной стороне треугольника, пересекающей две другие стороны, то она делит две стороны в одинаковом соотношении.
In ∆ABC, если DE || BC и пересекает AB в D и AC в E, затем
AD AE
—- = ——
DB EC

Доказательство теоремы Фалеса:

Если линия проводится параллельно одной стороне треугольника и пересекает две другие стороны в двух разных точках, то она делит две стороны в одинаковом соотношении.

Дано: In ∆ABC, DE || BC и пересекает AB в D и AC в E.

Докажите, что: AD / DB = AE / EC

Строительство: Присоединитесь к BC, CD и нарисуйте EF ┴ BA и DG ┴ CA.

)
Заявления Причины
1) EF ┴ BA 1) Конструкция
2) EF — высота ∆ADE и ∆DBE2 Определение
3) Площадь (∆ADE) = (AD .EF) / 2 3) Площадь = (Базовая высота) / 2
4) Площадь (∆DBE) = (DB.EF) / 2 4) Площадь = (Базовая высота) / 2
5) (Площадь (∆ADE)) / (Площадь (∆DBE)) = AD / DB 5) Разделить (4) по (5)
6) (Площадь (∆ADE)) / (Площадь (∆DEC)) = AE / EC 6) То же, что указано выше
7) ∆DBE ~ ∆DEC 7 ) Оба ∆s находятся на одном основании и
между одинаковыми || линий.
8) Площадь (∆DBE) = площадь (∆DEC) 8) Если два треугольника похожи, их
площади равны
9) AD / DB = AE / EC 9) От (5), (6) и (7)

Некоторые решенные примеры:
1) На данном рисунке PQ || MN.Если КП / ПМ = 4/13 и КН = 20,4 см.
Найдите KQ.

Решение:
In Δ KMN,

PQ || MN

∴ KP / PM = KQ / QN (по теореме BPT)

⇒ KP / PM = KQ / (KN — KQ)

⇒ 4/13 = KQ / (20,4 — KQ)

⇒ 4 (20,4 — KQ) = 13 KQ (перекрестное умножение)

⇒ 81,6 — 4KQ = 13 KQ

⇒ 17KQ = 81,6

⇒ KQ = 81,6 / 17

KQ = 4,8 см.

———————————————— ——————
2) На рисунке ниже DE || BC.Если AD = x см, DB = x-2 см,
AE = x-1 см, найдите значение x.

Решение:
В треугольнике ABC,

DE || BC

AD / DB = AE / EC (по основной теореме пропорциональности)

⇒ x / (x — 2) = (x + 2) / (x — 1)

⇒ x (x — 1) = (x — 2) (x + 2) (перекрестным умножением)

⇒ x 2 — x = x 2 — 4

⇒ -x = -4

∴ x = 4


Сходство в треугольниках

• Сходство в геометрии
• Свойства подобных треугольников
• Основная теорема пропорциональности (теорема Фалеса)
• Обращение к основной теореме пропорциональности
• Теорема о биссектрисе внутреннего угла
• Теорема о биссектрисе внешнего угла
• Доказательства основной пропорциональности
• Критерии Сходство треугольников
• Среднее геометрическое количество похожих треугольников
• Области двух похожих треугольников

Домашняя страница

Covid-19 повлиял на физическое взаимодействие между людьми.

Не позволяйте этому влиять на ваше обучение.

Теорема Фалеса: доказательство | Справка по геометрии

На сегодняшнем уроке мы докажем теорему Фалеса — вписанный угол, который образует диаметр окружности, всегда является прямым углом, используя сумму углов в треугольнике.

Фалес Милетский был греческим математиком, работа которого предшествовала работе Евклида и Пифагора.

С его именем связан ряд теорем. Одна из них — это теорема о пересечении для соотношений между линейными сегментами, созданными, когда две параллельные прямые пересекаются двумя пересекающимися линиями, и здесь мы докажем другую, связанную с вписанными углами.

Теорема утверждает, что любой вписанный угол в окружность, которая образует диаметр, является прямым углом.

Задача

CB — диаметр окружности с центром O, а A — точка на окружности окружности. Покажите, что m∠CAB = 90 °

Стратегия

Один из способов доказать это — рассматривать это как частный случай теоремы о вписанном угле: центральный угол равен удвоенному вписанному углу, который образует ту же дугу; диаметр разделяет круг (который по определению равен 360 °) на две равные половины, поэтому его центральная англоязычная мера составляет 180 °, а вписанный угол, который образует его, должен быть вдвое меньше, или 90 °.

Но поскольку работа Фалеса предшествовала Евклиду (который доказал общую теорему для вписанных углов), давайте докажем ее напрямую, не полагаясь на теорему о вписанных углах.

Соедините центр O с точкой A. OA — это радиус, а также OB и OC. Все радиусы равны, поэтому два созданных нами треугольника — ΔOAC и ΔOAB — являются равнобедренными треугольниками. И, согласно теореме о базовом угле, их базовые углы равны. Обозначим базовые углы ΔOAB как ‘α’, а углы ΔOAC ‘β’.

Сумма углов в треугольнике равна 180 °.Итак, α + α + β + β = 180 °, 2 * (α + β) = 180 °, поэтому α + β = 90 °. А поскольку ∠CAB = α + β, это прямой угол.

Доказательство

(1) OA = OB = OC = r // все радиусы равны
(2) ∠OCA≅∠OAC = β // Теорема об основном угле
(3) ∠OBA≅∠OAB = α // Теорема об основных углах
(4) α + α + β + β = 180 ° // сумма углов в треугольнике
(5) 2 * (α + β) = 180 ° // (4), распределительное свойство умножения
(6) α + β = 90 ° // (5), разделить на 2
(7) ∠CAB = mα + β // постулат сложения углов
(8) m∠CAB = 90 °

Прямоугольник Фалеса | Британника

Фалес Милетский процветал около 600 г. до н.э., и ему приписывают многие из самых ранних известных геометрических доказательств.В частности, ему приписывают доказательство следующих пяти теорем: (1) окружность делится пополам на любой диаметр; (2) углы основания равнобедренного треугольника равны; (3) противоположные («вертикальные») углы, образованные пересечением двух прямых, равны; (4) два треугольника равны (одинаковой формы и размера), если два угла и сторона равны; и (5) любой угол, вписанный в полукруг, является прямым углом (90 °).

Хотя ни одно из оригинальных доказательств Фалеса не сохранилось, английский математик Томас Хит (1861–1940) предложил то, что сейчас известно как прямоугольник Фалеса ( см. рисунок) в качестве доказательства (5), которое согласовывалось бы с то, что было известно в эпоху Фалеса.

Начиная с ∠ A C B , вписанного в полукруг диаметром A B , проведите линию от C через центр соответствующей окружности O так, чтобы она пересекала круг в точке D . Затем завершите четырехугольник, нарисовав линии A D и B D . Во-первых, обратите внимание, что линии A O , B O , C O и D O равны, потому что каждая из них является радиусом r окружности.Затем обратите внимание на то, что вертикальные углы, образованные пересечением линий A, , B, и C, , D , образуют два набора равных углов, как показано отметками. Применяя известную Фалесу теорему, теорему SAS (два треугольника конгруэнтны, если две стороны и включенный угол равны) дает два набора конгруэнтных треугольников: △ A O D ≅ △ B O C и △ D O B ≅ △ C O A .Поскольку треугольники совпадают, их соответствующие части равны: ∠ A D O = ∠ B C O , ∠ D A O = ∠ C B O , ∠ B D O = ∠ A C O и т. Д. Поскольку все эти треугольники равнобедренные, их базовые углы равны, а это означает, что есть два набора из четырех равных углов, как указано галочками.Наконец, поскольку каждый угол четырехугольника имеет одинаковую композицию, четыре угла четырехугольника должны быть равны — результат, который возможен только для прямоугольника. Следовательно, ∠ A C B = 90 °.

Теорема Фалеса и теорема о биссектрисе угла

Теорема Фалеса и теорема о биссектрисе угла

Введение

Фалес, (640 — 540 гг. До н.э.), самый известный греческий математик. и философ жил примерно в седьмом веке до нашей эры (до н.э.).Он обладал знаниями настолько, что он стал первым из семи мудрецов Греции. Фалес был первый человек, заявивший, что любая возникшая идея должна быть проверена научно и только тогда это можно будет принять. В этом аспекте он отлично справился исследования в области математики и астрономии и открыл много концепций. Он был признан за предоставление первого доказательства в математике, которую сегодня называют название «Основная теорема пропорциональности». Ее еще называют «теоремой Фалеса». назван в честь его первооткрывателя.


Открытие самой теоремы Фалеса является очень интересным сказка. Когда Фалес отправился в Египет, египтяне бросили ему вызов. определить высоту одной из нескольких великолепных пирамид, которые у них были построен. Фалес принял вызов и использовал подобие треугольников для определить то же самое, еще одно триумфальное применение геометрии. Поскольку X 0 , X 1 и H 0 являются Известно, можно определить высоту H 1 пирамиды.

Чтобы понять основную теорему пропорциональности или теорему Фалеса, давайте сделаем следующее: Мероприятия.


Теорема 1:

Основная теорема пропорциональности (BPT) или Thales теорема

Заявление

Прямая линия, проведенная параллельно стороне пересекающегося треугольника. две другие стороны делят стороны в таком же соотношении.


Проба

Дано: In Δ ABC , D — точка на AB и E — точка на AC .

Доказать:

AD / DB = AE / EC


Строительство:

Проведите линию DE || BC


Следствие

Если в Δ ABC , прямая DE параллельна BC , пересекает AB в точке D и AC в точке E , затем


Проба

In Δ ABC , DE || г. до н.э. .

, следовательно, => AD / DB = AE / EC (по Basic Теорема пропорциональности)



Верно ли и обратное к основной теореме пропорциональности? К рассмотрим, давайте сделаем следующую иллюстрацию.

Иллюстрация

Нарисуйте на ноутбуке угол XAY , как показано на рис. 4.31 и на луче AX , отметьте точки B 1 , B 2 , B 3 , B 4 и B так, что AB 1 = B 1 B 2 = B 2 B 3 = B 3 B 4 = B 4 B = 1 см.

Аналогично лучу AY , отметьте точки C 1 , C 2 , C 3 , C 4 и C , так что AC 1 = C 1 C 2 = C 2 C 3 = C 3 C 4 = C 4 C = 2 см, Присоединиться B 1 C 1 и BC

Обратите внимание, что

Аналогично присоединяется B 2 C 2 , B 3 C 3 и B 4 C 4 вы видите, что


Отсюда мы видим, что если прямая разделяет две стороны треугольник в таком же соотношении, то линия параллельна третьей стороне.

Таким образом, получаем следующую теорему, называемую обращением Теорема Фалеса.

Теорема 2:

Обратное к основной теореме пропорциональности

Заявление

Если прямая линия разделяет любые две стороны треугольника на одну и ту же соотношение, то линия должна быть параллельна третьей стороне.

Проба

Дано: In ABC ,

Доказать: DE || BC

Строительство Draw BF || DE



Теорема 3:

Теорема о биссектрисе угла

Заявление

Внутренняя биссектриса угла треугольника делит противоположная сторона внутри в соотношении соответствующих сторон, содержащих угол.

Проба

Дано: В ΔABC AD является внутренним биссектриса

Доказать:

Конструкция: Проведите линию через C параллельно AB . Расширить AD до линия встречи через C на E



Теорема 4:

Преобразование теоремы о биссектрисе угла

Заявление

Если прямая линия, проходящая через одну вершину треугольника, разделяет противоположная сторона внутри в соотношении двух других сторон, затем линия делит угол пополам внутри вершины.

Проба

Дано: ABC — треугольник. год нашей эры делит год до нашей эры в соотношении стороны, содержащие углы ∠ A , чтобы соответствовать BC на D .

То есть

Чтобы доказать: AD делит пополам ∠ A т.е. ∠1 = ∠2

Конструкция: Draw CE || DA . Расширьте BA до встречи по адресу E .



Пример 4.12

дюйм Δ ABC , если DE || BC , AD = x , DB = x 2, и EC = x −1 , затем найти длины стороны AB, и AC.

Решение

В Δ ABC имеем DE || BC .

По теореме Фалеса имеем


x / x -2 = ( x + 2) / ( x -1) дает x ( x — 1) = ( x — 2) ( x + 2)

Следовательно, x 2 x = x 2 — 4 так, x = 4

Когда x = 4, AD = 4, DB = x — 2 = 2, AE = x + 2 = 6, EC = x — 1 = 3

Следовательно, AB = AD + DB = 4 + 2 = 6, AC = AE + EC = 6 + 3 = 9.

Следовательно, AB = 6, AC = 9.

Пример 4.13

D и E — соответственно точки на сторонах AB и AC 9069 Δ AB203 = 5.6 см, AD = 1,4 см, AC = 7,2 см и AE = 1,8 см, показывают, что DE || BC .

Решение

У нас AB = 5,6 см, AD = 1,4 см, AC 7,2 см и AE = 1,8 см.

BD = AB AD = 5,6–1,4 = 4.2 см

и EC = AC AE = 7,2–1,8 = 5,4 см.


Следовательно, обратное основной теореме пропорциональности, мы имеем DE является параллельно г. до н.э.

Значит доказано.

Пример 4.14

На рис. 4.38, DE || AC и DC || AP .Докажи это .

Решение

Решение В ΔBPA мы имеем DC || AP. По базовой пропорциональности Теорема,

у нас


В Δ BCA имеем DE || AC . По базовой пропорциональности Теорема, имеем


Из (1) и (2) получаем, BE / EC = BC / CP => . Значит доказано.

Пример 4.15

На рисунке 4.39, AD является биссектрисой ∠ A . Если BD = 4 см, DC = 3 см и AB = 6 см, найти AC.

Решение

In Δ ABC , AD — биссектриса ∠ A

Следовательно, по теореме о биссектрисе угла


4/3 = 6 / AC

дает 4AC = 8.Отсюда AC = 9/2 = 4,5 см

Пример 4.16

На рисунке 4.40, AD является биссектрисой BAC , если AB = 10 см, AC = 14 см и BC = 6 см. Найдите BD и DC .

Решение

Пусть BD = x см, затем DC = (6– x ) см

AD является биссектрисой ∠ A

Следовательно, по теореме о биссектрисе угла


Следовательно, BD = 2.5 см, DC = 6 x = 6 2,5 = 3,5 см

Первые доказательства: Фалес и начало геометрии

Podcast: Download

Доказательная геометрия началась с Thales. Приписываемые ему теоремы заключают в себе два способа выполнения математических вычислений, предполагая, что идея доказательства могла исходить из двух источников: внимания к шаблонам и отношениям, возникающим в результате исследовательского построения и игры, или осознания того, что «очевидные» вещи могут быть продемонстрировано с использованием формальных определений и доказательства от противного.

Выписка

Как начались доказательства? Это похоже на головоломку с курицей или яйцом. Зачем кому-то садиться и говорить себе: «Я собираюсь доказать некоторые теоремы сегодня», если никто никогда не делал ничего подобного раньше? Как эта идея могла так неожиданно прийти в голову кому-то?

На самом деле, мы вроде как знаем ответ. Греческая традиция говорит нам, у кого был этот момент лампочки: у Фалеса. Примерно в году -600 или около того. За сотни лет до того, как у нас появились какие-либо прямые исторические источники по греческой геометрии.Но мы все еще более или менее знаем, что доказал Фалес. Более поздние источники рассказывают нам о Фалесе. История, возможно, смешана с легендами в такого рода сообщениях, но ключевые аспекты, вероятно, будут довольно надежными. Больше фактов, чем выдумок. Давайте проанализируем этот вопрос, вопрос о достоверности, немного более глубоко позже, но сначала давайте примем истории за чистую монету и посмотрим, как мы можем пережить создание дедуктивной геометрии, как это передано в этих греческих историях.

Итак, начнем: Какая первая теорема была доказана? Что стало той искрой, которая зажгла пожар аксиоматико-дедуктивной математики? Лучшее предположение, основанное на исторических свидетельствах, звучит так.Момент любви с первого взгляда, та теорема, открывшая нам глаза на силу математического доказательства, заключалась в следующем: диаметр разрезает круг пополам.

Довольно разочаровывающе, не правда ли? Какая неубедительная теорема. Это даже не теорема. Как можно влюбиться в геометрию, доказывая что-то столь тривиальное и очевидное?

Но не отчаивайтесь. Вообще-то, это приятно. Дело не в теореме, а в доказательстве.

Вот как это доказать. Предположим, что нет. Это будет доказательство от противного.Предположим, диаметр не делит круг на две равные половины. Хорошо, у нас есть линия, проходящая через середину круга, и она разрезана на две части. И мы предполагаем, что эти две части не совпадают. Возьмите один из кусочков и переверните его на другой. Как вы складываете омлет или креп. Мы предположили, что части не равны, поэтому, когда вы кладете одну на другую, они не совпадают. Так что должно быть какое-то место, где одна из двух частей будет выступать за другую.Теперь нарисуйте радиус в этом направлении от середины круга до места по периметру, где две половины не совпадают. Тогда один радиус длиннее другого. Но это означает, что дело было не в круге. Круг — это фигура, одинаково удаленная от середины во всех направлениях. Вот что значит быть кругом.

Итак, мы доказали, что две вещи несовместимы друг с другом: нельзя одновременно быть кругом и иметь несовпадающие половинки. Потому что, если у вас есть несовпадающие половинки, у вас также есть «неравные радиусы», а это значит, что вы не круг.

Итак, у круга должны быть равные половины. Бам. Теорема. Скучный результат, но великолепное доказательство. Или убедительное доказательство. Это доказательство, намекающее на новый мир.

Фалес, должно быть, чувствовал себя волшебником, который только что обнаружил, что обладает сверхспособностями. «Ого, ты можешь это сделать ?!» С помощью чистого рассуждения, вычеркивая последствия определения, можно без тени сомнения доказать, что определенные утверждения не могут быть неправильными? Это вещь? Это то, что можно сделать? Ух ты. Давай сделаем это со всем! Правильно?

Так вот как Фалес открыл доказательство.Насколько мы можем догадаться.

Несколько других теорем также приписываются Фалесу. Я хочу особо выделить одну, которая, как мне кажется, также является своего рода архетипом того, что из себя представляет математика.

Теорема, которую мы только что видели, о диаметре, разделяющем окружность пополам, идеально воплощает один из прототипов математических рассуждений. Вы могли бы назвать это парадигмой чистой математики. Логические следствия определений, доказательства от противного. Такие вещи. Доказательство Фалеса действительно поражает своей эстетикой.С тех пор мы делаем одно и то же снова и снова. Например, современный курс теории групп — это просто идея доказательства Фалеса, примененная, по сути, пятьсот раз.

Теперь я хочу взять еще один результат, приписываемый Фалесу, и я хочу утверждать, что он символизирует другой способ математической мысли. Это второй путь к доказательствам. Этот второй способ основан больше на игре, исследовании, открытии, чем на логике и определениях.

Пример, который я хочу использовать, чтобы показать это, — это то, что на самом деле часто называют просто «теоремой Фалеса».Который утверждает, что любой треугольник, возвышающийся на диаметре круга, имеет прямой угол. Другими словами, изобразите круг. Разрежьте его пополам диаметром. Теперь поднимите треугольник, используя этот диаметр как одну из его сторон, и третья вершина треугольника находится где-то на окружности. Так что это похоже на палатку, торчащую из диаметра. И это может быть асимметричный шатер, который больше направлен в ту или иную сторону. Независимо от того, как вы устанавливаете эту палатку, до тех пор, пока ее кончик находится в любой точке круга, угол между двумя стенами палатки в этой точке, на вершине, будет прямым углом, 90 градусов. .Это теорема Фалеса.

Как мог Фалес доказать эту теорему? К сожалению, мы не знаем этого, основываясь на исторических данных. Но давайте рассмотрим одну гипотезу, которая имеет контекстный смысл.

Мы должны представить, что Фалес каким-то образом наткнулся на доказательство. Мы не пытаемся объяснить, как кто-то может думать о доказательстве этой теоремы как таковой. Это неправильная точка зрения, потому что считается само собой разумеющимся, что в математике кто-то пытается что-то доказать. Нам нужно объяснить, откуда взялось это видение, чтобы доказать все в геометрии.Как мог кто-то случайно наткнуться на теорему Фалеса и в результате этой случайности осознать идею дедуктивной геометрии?

Действительно, теорема Фалеса сама по себе не слишком интересна или важна. Если бы у вас было видение систематического доказательства всей геометрии, почему бы вам начать с этой теоремы или сделать ее центральной частью, как якобы сделал Фалес? Ты бы не стал.

Теорема Фалеса интересна не в том, что это был один из первых результатов, к которому математики применили дедуктивное доказательство.Скорее, интересно то, что математики случайно наткнулись на саму идею доказательства.

Есть история о Фалесе, который упал в колодец, потому что он настолько увлекся астрономическими рассуждениями, что забыл о своем окружении. У Платона записано: «Когда он изучал звезды и смотрел вверх, он упал в яму. Поскольку ему так хотелось познать вещи в небе, он не мог видеть то, что было перед ним у самых его ног.”

Может быть, это легенда, но открытие теоремы Фалеса должно было быть чем-то вроде этого. Обнаружение математического доказательства должно было быть похоже на падение в яму. Вы смотрите в одном направлении и, бум, внезапно обнаруживаете, что сначала случайно врезаетесь лицом в эту совершенно не связанную новую вещь, о существовании которой вы даже не подозревали.

Как могла быть теорема Фалеса такой? Что делает теорему Фалеса среди всех мировых теорем особенно благоприятной для такого рода случайного открытия доказательства?

Вот моя гипотеза.В наш век невинности, прежде чем кто-либо знал что-либо о доказательствах, людям все еще нравились формы. У них были линейка и компас. Они использовали эти инструменты для измерения полей и прочего, но им также нравилась их эстетика.

Они игрались с линейкой и компасом. Играем с фигурами. После пяти минут игры с компасом вы узнаете, как нарисовать правильный шестиугольник. Помните? Вы, наверное, делали это в детстве. Нарисуйте круг, а затем, не меняя отверстия компаса, проведите циркулем по окружности.Умещается ровно шесть раз. Очень приятная форма.

Мы точно знаем, что люди делали это до Фалеса. В мозаиках Месопотамии есть шестиугольные мозаичные узоры, датируемые примерно -700 годом.

Додекаэдры — еще одна из таких вещей. Додекаэдр похож на те двенадцатигранные кости, которые вы используете в Dungeons and Dragons и тому подобном. До-дека-эдр, буквально: двухсторонний. Другими словами, так двенадцать сторон. Двенадцать граней, каждое из которых представляет собой правильный пятиугольник.Эти вещи есть в археологических записях. Люди делали их из камня и бронзы. Было обнаружено несколько десятков древних додекаэдров, самые старые из них были найдены еще до Фалеса. Возможно, они использовались для пророческих целей, например, карты Таро или что-то в этом роде. А может, для настольных игр, кто знает?

В любом случае, я хочу сказать, что людей интересовали геометрические узоры для различных целей: художественных, культурных и так далее. Не только для измерения полей для налоговых целей. И они явно работали с такими инструментами, как линейка и компас, чтобы делать эти вещи.

К теореме Фалеса легко прийти, просто играя с линейкой и циркулем, пытаясь нарисовать красивые вещи. Начнем с прямоугольника. Нарисуйте его диагонали. Поместите стрелку циркуля в место пересечения, прямо в середину прямоугольника. Установите перо циркуля в один из углов прямоугольника. Теперь крутите его. У вас получится круг, который идеально и плотно прилегает к прямоугольнику.

Но посмотрите, что получилось. Диагональ прямоугольника становится диаметром круга.А торчащие из него прямоугольники — это как раз те треугольники-шатры, о которых говорит Теорема Фалеса. Это внезапно делает теорему очевидной.

Почему теорема Фалеса верна? Почему любая из этих «палаток», поднятых на диаметре круга, имеет прямой угол? Потому что он образован прямоугольником. Любая такая палатка представляет собой половину прямоугольника. Это мощный сдвиг в перспективе. Глядя на треугольник таким образом, мы обнаруживаем скрытые отношения, скрытый порядок в природе вещей.Определенные углы всегда должны быть прямыми, так сказать, по некой метафизической необходимости. Наши глаза были открыты, возможно, впервые, на существование такого рода потребностей, этих видов скрытых отношений, которые существует, чтобы мыслящий человек мог раскрыть.

Итак, ключ к этому сдвигу перспективы, что треугольник «на самом деле» является половиной прямоугольника. Предположим вместо этого, что мы застряли в точке зрения, что мы смотрим на треугольник, вписанный в круг.Тогда те ассоциации и идеи, которые нам напрашиваются, не столь полезны для доказательства этой теоремы. С этой точки зрения, если бы вы искали доказательства, что бы вы сделали? Может быть, вы, например, соедините середину круга с концом треугольника. Итак, теперь у вас есть два меньших треугольника. Что ты собираешься с ними делать? Что-нибудь с суммами углов и так далее? Или, может быть, у вас возникнет соблазн опустить перпендикуляр вместо вершины треугольника, а затем вы сможете использовать теорему Пифагора о двух полученных маленьких треугольниках.

Такие вещи не то, что мы хотим. Подобные подходы быстро становятся слишком техническими. Помните, это должно было быть началом геометрии. Вы не должны использовать несколько предыдущих результатов для доказательства. Это должно быть доказательство из первых принципов. Доказательство перед всеми другими доказательствами.

Идея о том, что треугольник «на самом деле» является половиной прямоугольника, отличается. Это меняет то, как мы смотрим на диаграмму. Это меняет акценты. Это меняет то, что мы считаем основным.Теперь прямоугольник идет первым, треугольник — вторым, а круг — последним. С этой точки зрения теорема на самом деле вовсе не касается кругов, так сказать. Круг — это просто вторичный артефакт.

В этом доказательстве мы как художники. Мы отошли от холста, наклонили голову и увидели это прозрение. И прозрение стало возможным благодаря тому, как мы раньше играли с этими идеями. Мы просто играли с линейкой и циркулем, мы исследовали треугольники, прямоугольники и круги с непредубежденной любовью.Из этой пьесы рождаются такие прозрения, как теорема Фалеса. В этом контексте вдохновение приходит естественно.

В отличие от других скучных доказательств, на которые я ссылался, которые были основаны на разрезании треугольника и бросании в него книги: суммы углов, теорема Пифагора, все, что мы можем придумать. Это скучный подход, подход грубой силы. Ему не хватает того эстетического вдохновения, того прозрения, раскрывающего истинную природу треугольника и его второй половины, с которой ему суждено было воссоединиться.

Геометрия не могла начаться с такого рода практических доказательств, потому что они имеют смысл только после того, как есть книга по геометрии для начала. Но геометрия могла начаться с доказательства типа прозрения. Таким образом, кто-то вроде Фалеса мог прийти к идее доказательства, играя с линейкой и компасом.

Возможно, вы знакомы с «Плачом Локкарта»: отличным эссе о том, что не так с математическим образованием. Иди и прочитай, он доступен в Интернете.Интересно, что Локхарт использует именно этот пример, чтобы доказать свою точку зрения. Он описывает, как его ученики открыли теорему Фалеса, в основном так, как я говорю, что Фалес мог это сделать. Он также красноречиво показывает, насколько это удовлетворяет гораздо больше, чем сухое доказательство по книге.

Недаром в этом вопросе история и образование идут рука об руку. Доказательство должно было начаться с убедительного эстетического опыта или вау-момента. В то время другого пути не было. Заставить Фалеса запоминать факты для экзамена было некому.Открытие заставило его ценить математику. Если мы хотим развить внутреннюю мотивацию у наших студентов, неплохо было бы в первую очередь подумать, что заставило людей полюбить эти идеи. Первая любовь всегда чистейшая и невинная. Современные учебники похожи на браки по договоренности, навязанные студентам. Но в истории всегда есть настоящая история любви.

Тем не менее, несмотря на все это, вы все равно можете подумать, что теорема Фалеса немного скучна. Что-то всегда есть под прямым углом.Ну и что? Какая разница?

Как я пытался утверждать, Фалеса и его современников, вероятно, впечатлила не сама по себе теорема, а, скорее, идея о том, что теоремы и доказательства вообще существуют. Есть скрытые истины, которые можно раскрыть с помощью рассуждений. Замечательный.

Но на самом деле интересна и сама теорема. Позвольте мне показать вам кое-что интересное, что вы можете сделать с помощью теоремы Фалеса.

Существует древняя легенда о царице Дидоне.Дочь царя Тира, крупного города в древности. Вы все еще можете увидеть руины этого древнего города на территории современного Ливана. В какой-то момент Дидоне пришлось бежать из-за придворных интриг. Убийства и предательства и так далее. Поэтому она берет с прикроватной тумбочки пару диадем, может быть, сундук с золотом, который отложила на дождливый день, и торопливо уходит в ночь. С едва ли в мире остался друг.

Ей нужно пройти весь путь до нынешнего Туниса, находящегося за тысячи километров, и попытаться как-нибудь начать все сначала, как приличествует королевской семье.Используя свой сундук с сокровищами, она заключает сделку, чтобы купить землю. Как гласит легенда, столько земли, сколько она может покрыть шкурой быка. Она разрезала бычью шкуру на тонкие полоски и связала их вместе, и что теперь? Итак, теперь у нее есть эта длинная веревка, которую она может использовать как своего рода забор, чтобы изолировать землю, которую она хочет.

Но какой формы сделать? Квадрат, прямоугольник, треугольник? Нет. Дидоне виднее. Возможно, ее королевское образование включало математику. Сделайте это круглым. Это лучший способ.Круг имеет максимальную площадь среди всех фигур с заданным периметром. Или, в данном случае, поскольку она была у океана: полукруг, с другой стороны береговой линией как естественной границей.

Давай докажем это. Что полукруг — лучший выбор. Я собираюсь доказать это с помощью противоречия: предположим, что кто-то построил забор на участке, который не является полукругом; Затем я могу показать, как это сделать лучше: как переместить забор так, чтобы территория стала еще больше, без добавления забора.

Хорошо, у вас есть береговая линия, это прямая линия. И из одной точки на берегу, идя вглубь суши, у вас есть этот забор, который затем снова спускается и снова встречается с берегом в какой-то другой точке. Так что вместе с береговой линией он закрывает определенную территорию.

Предположим, эта форма не является полукругом. Если бы это был полукруг, применима теорема Фалеса. И он сказал бы вам, что этот угол, то, что я назвал углом палатки, в любой точке вдоль забора будет прямым углом.Итак, если фигура не является полукругом, на заборе должна быть точка, где этот угол не является прямым.

Я говорю, что если сделать этот угол прямым, то увеличится площадь покрытия. Вы можете представить это так. Итак, у вас есть форма, окруженная забором: представьте, что у вас есть вырез из картона. А по периметру у вас есть отмеченная точка, где угол палатки не является прямым ангелом. Итак, на вашем картоне нарисован этот треугольник: треугольник, состоящий из прямой береговой линии с одной стороны и двух линий от его концов, идущих вверх, чтобы встретиться в точке палатки по периметру.

Давайте вырежем этот треугольник из картона. Итак, у вас остались две части: любые части, которые торчали из сторон треугольника. Теперь переместите эти две части так, чтобы угол палатки получился прямым. Это означает перемещение конечных точек вдоль береговой линии. Перемещая две точки на береговой линии, вы меняете угол, под которым встречаются две картонные части. Две картонные части встречаются в одной точке, точке палатки, и это похоже на петлю, которая может открываться или закрываться на больший или меньший угол.Таким образом, вы перемещаете эти штуки, пока угол поворота не станет 90 градусов.

Обратите внимание, что вы не меняли периметр таким образом. Вы только что переместили такое же количество забора.

Но вы действительно увеличили огражденную площадь. Потому что, если у вас есть две палки фиксированной длины, и вы хотите сделать из них самый большой треугольник, лучший способ — сделать угол между ними прямым. Это интуитивно понятно. Вы знаете, что площадь треугольника равна основанию, умноженному на высоту, больше двух.Итак, если одна из ваших палочек является основанием, то для увеличения площади вы хотите максимизировать высоту, то есть перпендикулярную высоту, идущую вверх от основания, что, очевидно, достигается путем направления другой палки прямо вверх под прямым углом.

Итак, это доказывает, что для любого ограждения, отличного от полукруга, вы можете сделать лучший. Вы можете передвинуть забор и увеличить площадь. Так что полукруг — лучшее решение, а все остальные менее хороши.

Не знаю, сможете ли вы вообразить все это.Но, может быть, позже попытайтесь реконструировать этот аргумент для себя. Это действительно очень интуитивно понятно и красиво.

Так в чем же тогда мораль этой истории? Математически это ответ на вопрос «ну и что?» вопрос относительно теоремы Фалеса. Это могло показаться достаточно скучной теоремой, но здесь мы видим ее в действии красивым и неожиданным образом, как ключевой ингредиент в этом доказательстве о том, как оградить землю. Кто бы мог предвидеть это?

Это говорит о том, что у математики есть своего рода «снежный ком» или аспект самооплодотворения.Теорема Фалеса, в чем дело? Просто скучное наблюдение о треугольнике в круге. Может показаться, что это не так уж много. Но одно ведет к другому. Как только вы примете теорему Фалеса, вы начнете видеть ее в других, неожиданных местах. Как эта проблема о районе. Вы бы не подумали, что это связано, но чем больше вы занимаетесь математикой, тем больше связей вы обнаруживаете.

Выберите любую теорему, какой бы скучной она ни была, например, теорема Фалеса, и вы сможете найти эти удивительные вещи, в которых скучная теорема на самом деле является ключевым моментом, открывающим совершенно новые способы осмысления, казалось бы, не связанных между собой проблем.Это для вас математика. Неудивительно, что греки прижились как жук, как только они сдвинулись с мертвой точки. В один момент вы натыкаетесь на какой-то случайный результат, вроде теоремы Фалеса, а в следующий момент вы знаете, что видите математику повсюду.

Такова математическая мораль истории. Теперь мы должны вернуться и сказать кое-что об исторической стороне всего этого. Что мы на самом деле знаем о Фалесе и его теоремах, королеве Дидоне и всем остальном? Сколько истории и сколько легенд?

Если мы начнем с Дидоны, то эта история используется в основном через Вергилия.Энеида, известная эпическая поэма. Это было написано во времена Римской империи, примерно в -20 году. Но это относится к историческим или предположительно историческим событиям, которые произошли еще до Фалеса, может быть, за два столетия до Фалеса, то есть примерно 800-х годов. У нас есть версия Верджила, это то, что до нас дошло, но он просто ворует старую историю. Эти вещи существовали веками в греческой культуре, в различных литературных и исторических пересказах, которые сейчас утеряны.

Совершенно правдоподобно, что действительно существовала такая историческая королева, которая действительно покинула свой королевский дом в Тире и действительно приземлилась на северных берегах Африки, где она основала это новое поселение, которое должно было стать великим городом Карфаген. .Может быть, она даже сделала полукруглыми городские стены, кто знает? Совершенно очевидно, что она могла хотеть минимизировать периметр по какой-то причине, и что она могла знать, что полукруглая форма была оптимальной для этой цели.

Но в то время не было бы никаких математических доказательств этого, подобных тому, которое я обрисовал выше. Приведенное мной доказательство принадлежит Якобу Штайнеру в начале 19 века. Со времен Греции у нас есть другое доказательство этого результата.Так что они, конечно, очень хорошо знали результат, что полукруг является оптимальным, если, возможно, не то конкретное доказательство, которое я предложил.

Если история царицы Дидоны что-то говорит об истории математики, она, вероятно, больше всего не освещает ни время, когда произошли события, около -800, ни время, когда были написаны источники, которые у нас есть, около года 0. Но, может быть, это так. что-то говорит о промежуточных веках, когда история могла бы быть передана и переработана.

Рассказ был как бы маринован в греческой культуре.Возможно, они были теми, кто придал этому математический оттенок. Подходит для обуви: греки ценили мудрых, аристократических, хорошо образованных правителей, которые разрабатывают рациональную политику для общего блага, основанную на разуме и математике. Может быть, они позволили этим идеалам окрасить то, как они пересказывают историю королевы Дидоны и ее круглого города.

С этой точки зрения мы могли бы также предположить, что к тому времени, когда Вергилий придет и напишет римскую версию истории, это понимание математики уже не то, чем было раньше.На самом деле Верджил не раскрывает математический аспект оптимизации истории. Дидона — всего лишь второстепенный персонаж. Его эпопея повествует об Энее, который выполняет поиски, которые в конечном итоге приведут к основанию Рима.

Эней терпит кораблекрушение и выносится на берег в Карфагене, круглом городе Дидоны. Дидона влюбляется в него, но он не отвечает на ее любовь. Он уплывает, и Дидона убивает себя из-за разбитого сердца. Моррис Клайн завершает рассказ: «Итак, неблагодарный и невосприимчивый человек с жестким умом стал причиной потери потенциального математика.Это был первый удар по математике, нанесенный римлянами ». Конечно, есть еще много всего, откуда это взялось.

Можно рассматривать эту историю как символ этого перехода от мудрых философов-королей (или в данном случае королев) греческого мира, которые лелеяли математику и использовали ее для улучшения мира. Переход от этого к бессердечному римлянину, который думает только о себе и не заботится о теореме Фалеса. В греческом мире ботаники-математики считались привлекательными, но почему-то эти невежественные римляне явно не думали, что королева-геометрист вообще может быть подружкой.

Итак, история о Дидоне, круглом городе, доказательстве оптимизации и всем остальном очень интересна с точки зрения более широких математических и культурных точек, с которыми она связана, но сама по себе она не является историей как таковой.

С Thales все иначе. Это больше факт, чем легенда. Насколько мы можем определить, Фалес действительно доказал, что диаметр делит окружность пополам, скорее всего, с помощью доказательства, рассмотренного выше.

Источники, которые у нас есть для этого, далеки от совершенства.В первую очередь Прокл, писавший примерно в 450 году, то есть через тысячу лет после жизни Фалеса. Такого рода запоздалые источники попадают в заблуждение. У них нет авторитета сами по себе. Прокл был никем. Его собственное понимание истории и математики очень плохо. Посредственный мыслитель, посредственный ученый, живущий в посредственном возрасте.

Это те источники, которые у нас есть. По сути, столь же авторитетный, как факт, который вы читаете на обратной стороне коробки с хлопьями или чего-то в этом роде.

Но есть надежда.Во времена своей славы Греция была просто выдающейся интеллектуальной культурой. И кое-что, например, о Фалесе, можно проследить до тех пор, что делает его весьма достоверным. Ученик Аристотеля Евдем написал историю геометрии. Увы, этого уже нет. Возраст невежества игнорировал это, и теперь его нет. Но какая бы это была работа.

Эти люди знали, что делали. Более поздние люди, подобные Проклу, похожи на некоторых онлайн-рандомов, которые публикуют недоработанные идеи в блоге или плохо информированные комментарии в Facebook.Вот насколько они заслуживают доверия.

Но люди вроде Евдема — совсем другое дело. Это больше похоже на первоклассного ученого в исследовательском учреждении со всей инфраструктурой, о которой можно только мечтать: библиотеки, чрезвычайно знающие и умные коллеги с широким спектром знаний, широкая финансовая и культурная поддержка со стороны общественности и политиков и т. Д. . «История геометрии» Евдема могла бы стать настоящей книгой «University Press», отрецензированной до зубов и с красивой аннотацией Аристотеля в суперобложке.

Такие люди, как Евдем, не занимались распространением случайных сплетен и непроверенных фактов, потому что они звучат круто. Они были настоящими учеными и интеллектуалами.

И действительно, многие сведения о Фалесе можно проследить до этого потерянного источника. Когда Прокл говорит, что Фалес был первым, кто доказал, что круг делится пополам по его диаметру, источником этого является Евдем. Следовательно, это очень достоверно. Это случилось с Фалесом на самом деле. На самом деле часть о диаметре, разделяющем окружность пополам, более достоверна, чем часть о теореме Фалеса.Была ли теорема Фалеса на самом деле теоремой Фалеса? Может быть. Но мы не можем проследить конкретно эту часть до лучших источников. В отличие от диаметра пополам одной и некоторых других деталей. Но с точки зрения контекста это имеет смысл.

Рассказы о Фалесе и происхождении геометрии, очевидно, были хорошо известны не только специализированным ученым, но и широкой афинской публике. Драматург Аристофан несколько раз в своих пьесах использует имя Фалеса как символ геометрии. Так же, как сегодня, можно использовать имя Эйнштейна, например, чтобы вызвать образ ученого.У Аристофана один говорящий в диалоге говорит: «Этот человек — Фалес». Это означает, что человек — геометр. Очевидно, можно было ожидать, что театральная публика в классических Афинах поймет это упоминание. Каждый образованный человек знал бы о Фалесе и истоках геометрии.

На самом деле, общественное уважение к геометрии и ее истории было очевидно настолько велико, что Аристофан даже заставил одного из своих персонажей оплакивать ее как чрезмерную, говоря: «Почему мы продолжаем восхищаться старым Фалесом?» Какое время было бы жить.Когда драматургам приходилось решать такие проблемы, как чрезмерное уважение и интерес к математике среди широкой публики.

Добавить комментарий

Ваш адрес email не будет опубликован. Обязательные поля помечены *